Neuropathology Flashcards

1
Q

A patient has a nevus flammeus on one side of the face. Radiographs of the head show curvilinear
calcifications ipsilaterally. Calcium deposits in this patient’s brain are most likely to be found
Answers:
A. In the Choroid Plexus
B. In the Basal ganglia
C. In the Periventricular Region
D. In the Cortex and subcortex
E. In the Corpus Callosum

A

In the Cortex and subcortex

Discussion:
Sturge-Weber is associated with a “tram track” appearance of cortical and subcortical
calcifications. In a child with developmental delay presenting with seizures, macrocephaly,
presence of port-wine stains on the body, and imaging features of ‘”tram-track” pattern of
calcifications, parenchymal volume loss, enlarged choroid plexus and calvarial hyperostosis, the
diagnosis of Sturge-Weber syndrome should be made. Treatment includes seizure control, with
surgical resection being restricted to refractory cases.
Corpus callosum lipomas can be associated with calcification.
Basal ganglia calcifications are associated with Fahr’s disease.
Periventricular calcifications are associated with congenital CMV infection.
Choroid plexus calcification may be physiologic or associated with neurofibromatosis.
References:
Ragupathi S, Reddy AK, Jayamohan AE, et al “Sturge-Weber syndrome: CT and MRI illustrations.”
Case Reports 2014;2014:bcr2014205743.
Thomas-Sohl K, Vaslow D, Maria B. “Sturge-Weber syndrome: a review.” Pediatr Neurol
2004;30:303–10

How well did you know this?
1
Not at all
2
3
4
5
Perfectly
2
Q

Which of the following abnormalities is most likely in Creutzfeldt-Jakob disease?
Answers:
A. Pick cells and ballooned neurons
B. Tufted astrocytes and oligodendroglial coiled bodies
C. Neurofibrillary tangles and neuropil threads
D. α-Synuclein aggregates and glial cytoplasmic inclusions
E. Vacuolization in neuropil with gliosis

A

Vacuolization in neuropil with gliosis

Discussion:
Creutzfeldt–Jakob disease (CJD) is a human transmissible spongiform encephalopathy that
causes CNS degeneration. Prions (PrP), the causative agents of CJD, are composed of misfolded
proteins that can self-replicate and cause normally folded proteins to also misfold. Typically,
symptoms include progressive dementia, ataxia, and myoclonus. Characteristic neuropathological
features include neuronal loss, spongiform degeneration, reactive astrogliosis, and deposition of
misfolded PrP species. EEG can have characteristic generalized periodic sharp wave pattern. CSF
can demonstrate elevated 14-3-3 protein. MRI often shows high signal intensity in the basal
ganglia nuclei. Definitive diagnosis is through biopsy. The classic histologic appearance is
spongiform change in the gray matter: the presence of many round vacuoles from one to 50
micrometers in the neuropil, in all six cortical layers or with diffuse involvement of the cerebellar
molecular layer. These vacuoles appear glassy or eosinophilic and may coalesce. Neuronal loss
and gliosis are also seen. Plaques of amyloid-like material can be seen in the neocortex in some
cases of CJD.
Neurofibrillary tangles and neuropil threads can be seen in Alzheimer’s disease. Pick cells with
ballooned neurons are seen with Pick’s disease. Tufted astrocytes and oligodendroglial coiled
bodies are seen in Progressive Supranuclear Palsy. α-Synuclein aggregates and glial cytoplasmic
inclusions are seen in Lewy body disorders and Multiple System Atrophy.
References:
ee J, Hyeon JW, Kim SY, Hwang KJ, Ju YR, Ryou C. Review: Laboratory diagnosis and
surveillance of Creutzfeldt-Jakob disease. J Med Virol. 2015;87(1):175-186.
Pubmed Web link: https://pubmed.ncbi.nlm.nih.gov/24978677/
Liberski PP. Spongiform change–an electron microscopic view. Folia Neuropathol. 2004;42 Suppl
B:59-70.
Pubmed Web link: https://pubmed.ncbi.nlm.nih.gov/16903142

How well did you know this?
1
Not at all
2
3
4
5
Perfectly
3
Q

A 40-year-old woman presents with a right thoracic radiculopathy refractory to medical management. Physical examinaton shows no abnormalities. Activity, deep breathing, and coughing worsen the sharp pain in her right chest wall. A selective right T10 thoracic nerve block relieves the pain for two days. MR imaging of a right T10 mass is shown in Figures 1 and 2. Surgical excision results in relief of the thoracic radiculopathy. Based on MR imaging and the histopathology shown in Figure 3, which of the following is the most likely diagnosis?
Answers:
A. Arachnoid cyst
B. Synovial cyst
C. Herniated disc
D. Osteoid osteoma
E. Schwannoma

A

Synovial cyst

Discussion:
Based on the clinical history, imaging, and histology, the diagnosis is that of a synovial cyst.
Synovial cysts are benign herniations of the synovium of a facet joint. They occur due to
degenerative changes, and are asymptomatic except for when they impinge on a nerve root,
where they can cause radiculopathy, numbness, and weakness. T2 weighted MRI will be high
signal (bright), indicative of fluid within the cyst. This MRI shows that it is coming off the facet joint
into the foramen, causing the radiculopathy due to compression. Pathology shows fluid filled
synovium with an epithelial lining and myxoid degeneration. Disc herniations should be part of the
differential diagnosis of a synovial cyst, but are hypointense on T2 weighted imaging.
Histopathology should show only connective tissue without cystic changes. Osteoid osteomas are
primary bone lesions, do not invade into the spinal canal, are hypointense on T2-weighted
imaging, and will not have cystic changes on histopathology. Schwannomas are nerve-sheath
tumors and can be hyperintense on T2 weighted MRI. However, histopathology should not show
cysts but cellular areas (Antoni A), nuclear palisading (Verocay bodies), and hypocellular areas
(Antoni B). Arachnoid cysts are benign outpouching of the arachnoid and are normally incidental
on imaging, rarely causing symptoms. They are similar to synovial cysts on MRI, but
histopathology would not show an epithelial lining of the cysts.
References:
Atlas SW, ed. Magnetic Resonance Imaging of the Brain and Spine. 4th ed. Vol. 1 and 2. Wolters
Kluwer Health/Lippincott, Williams & Wilkins: Philadelphia, PA; 2009:1-1890.2.
Burger PC, Scheithauer BW, Kleinschmidt-DeMasters BK,et al. Diagnostic Pathology:
Neuropathology. Amirsys Publishing: Salt Lake City, UT. 2012

How well did you know this?
1
Not at all
2
3
4
5
Perfectly
4
Q

A 32-year-old woman is evaluated because of headaches and has brought an MR image of the
brain that was obtained by her general practitioner. T2-weighted (Figure 1) and contrast-enhanced
T1-weighted (Figure 2) axial MR images are shown. Although not present on these images, up to
30% of all patients harboring this pathology will have a coincident lesion known as which of the
following?
Answers:
A. Developmental Venous Anomaly
B. Bilateral Vestibular Schwannoma
C. Optic Nerve Glioma
D. Subependymal Giant Cell Astrocytoma
E. Pulmonary Arteriovenous Fistulae

A

Developmental Venous Anomaly

Discussion:
Up to 30% of all patients harboring a cavernous malformation will have a coincident lesion known
as a developmental venous anomaly, also known as a venous angioma. The “popcorn”
appearance of the lesion demonstrates the multiple ages of hemorrhage associated with the lesion
and the dark T2 rim suggests hemosiderin deposition, strongly supporting this diagnosis.
Bilateral vestibular schwannoma is associated with NF2, while optic nerve gliomas are associated
with NF1. Subependymal giant cell astrocytomas are seen in patients with tuberous sclerosis and
pulmonary AV fistula are seen with Osler-Weber-Rendu syndrome, a genetic cause of cerebral AV
malformations.
References:
Idiculla PS, Gurala D, Philipose J, Rajdev K, Patibandla P. Cerebral Cavernous Malformations,
Developmental Venous Anomaly, and Its Coexistence: A Review. Eur Neurol. 2020;83(4):360-368.
Pubmed Web link: https://pubmed.ncbi.nlm.nih.gov/32731220/
Awad IA, Polster SP. Cavernous Angiomas: Deconstructing a Neurosurgical Disease. J Neurosurg.
2019 Jul 1;131(1):1-13. Pubmed Web link: https://pubmed.ncbi.nlm.nih.gov/31261134/

How well did you know this?
1
Not at all
2
3
4
5
Perfectly
5
Q

A 29-year-old woman is brought to the emergency department because of sudden onset of neck pain, nausea, and vomiting immediately after riding on a rollercoaster. An MR image and an angiogram are shown. Which of the following is the most likely diagnosis?
Answers:
A. Carotid Dissection
B. Ruptured Vertebral Artery Aneurysm
C. Ruptured MCA aneurysm
D. Pituitary Apoplexy
E. Vertebral Dissection

A

Vertebral Dissection

Discussion:
The catheter-based angiogram shown here displays a vertebral artery dissection. Vertebral artery
dissection is a rare cause of stroke in the general population; however, it represents one of the
more common causes of stroke in patients younger than 45 years of age. Neck distortion often
precipitates the dissection. The dissection of the artery may ultimately lead to a stroke. Patients
with connective tissue disorders are also at increased risk. Ehlers-Danlos syndrome is the most
common connective tissue disorder that can cause vertebral artery dissection.
On MRI, fat suppressed T1 axial images may demonstrate a cervical arterial intramural hematoma,
or demonstrate a crescent sign of hyperintensity in the wall of the affected vessel as seen here.
Angiography is the gold standard and may identify a double lumen or an irregular lumen.
References:
Tiu C, Terecoasa E, Grecu N, Nistor R, Frangu S, Antochi F. Vertebral Artery Dissection: a
Contemporary Perspective. Maedica (Bucur). 2016 Jun;11(2):144-149. PMID: 28461834; PMCID:
PMC5394579. https://pubmed-ncbi-nlm-nih-gov.eresources.mssm.edu/28461834/
Britt TB, Agarwal S. Vertebral Artery Dissection. 2021 Aug 16. In: StatPearls [Internet]. Treasure
Island (FL): StatPearls Publishing; 2022 Jan–. PMID: 28722857. https://pubmed-ncbi-nlm-nihgov.eresources.mssm.edu/28722857/

How well did you know this?
1
Not at all
2
3
4
5
Perfectly
6
Q

A 52-year-old woman is evaluated for a two-year history of intermittent tingling paresthesias across
the top of her shoulders, into both hands and arms, and occasionally into her right leg. A T2-
weighted MR image of her cervical spine is shown. Which of the following is the most likely
diagnosis?
Answers:
A. Ependymoma
B. Hemangioblastoma
C. Cavernous Malformation
D. Astrocytoma
E. Acute Transverse Myelitis

A

Cavernous Malformation

Discussion:
The patient whose MR image with gadolinium is shown most likely has a Cavernous Malformation.
Although mostly sporadic, they can be associated with CCM1, CCM2, or CCM3 genes. The
imaging characteristics include the presence of a “popcorn” or “berry” sign with low T2 signal
intensity at the rim due to hemosiderin deposition. The “popcorn” rim suggests variable T1 and T2
signal from blood of varying ages.
Hemangioblastomas often occur in the posterior fossa and/or the spinal cord and have a nonenhancing cystic lesion with an avidly enhancing mural nodule. Ependymomas often occur in the
cervical spine from the cells lining the central canal and therefore have a central location with
hydromyelia from obstruction of the spinal fluid. Astrocytomas arise from the glial cells of the cord
itself, causing a bulbous appearance with patchy enhancement. Acute demyelinating processes do
not typically have a long course to presentation as this patient’s case. The imaging characteristics
of acute transverse myelitis show patchy enhancement.
References:
Mariano R, Flanagan EP, Weinshenker BG, Palace J. A practical approach to the diagnosis of
spinal cord lesions. Pract Neurol. 2018 Jun;18(3):187-200.
Pubmed Web link: https://pubmed.ncbi.nlm.nih.gov/29500319/
Beh SC, Greenberg BM, Frohman T, Frohman EM. Transverse myelitis. Neurol Clin. 2013
Feb;31(1):79-138. Pubmed Web link: https://pubmed.ncbi.nlm.nih.gov/23186897

How well did you know this?
1
Not at all
2
3
4
5
Perfectly
7
Q

The tumor in the contrast-enhanced T1-weighted MR image shown contains which of the following histologic features?
Answers:
A. Flexner-Winterstein Rosettes
B. Thin-walled capillary vessels, densely packed together with scarce parenchyma
C. Homer Wright Rosettes
D. Adamantinomatous types with distinctive epithelium that forms stellate reticulum, wet keratin, and basal palisades or a Papillary type with fibrovascular cores linked by nonkeratizing squamous epithelium
E. Perivascular pseudorosettes

A

Adamantinomatous types with distinctive epithelium that forms stellate reticulum, wet keratin, and basal palisades or a Papillary type with fibrovascular cores linked by nonkeratizing squamous epitheliu

Discussion:
The tumor in the contrast-enhanced T1-weighted MR image shown is classic for
craniopharyngiomas due to its cystic and solid components as well as its location in the sella and
suprasellar region. Craniopharyngiomas will frequently grow to involve the third ventricle,
hypothalamus, optic chiasm and pituitary gland. There are two main categories of
craniopharyngiomas. Adamantinomatous Craniopharyngioma has a bimodal peak for age at
presentation (5-15 and 45-60). On MRI they display cauliflower shape, frequently displaying
calcifications, enhancement, and/or cysts. On pathology, there is a distinctive epithelium that forms
stellate reticulum, wet keratin, and basal palisades.The papillary craniopharyngioma often
manifests in patients 40-55 years old. On MRI, they are mostly solid and rarely cystic. On
histology, there are fibrovascular cores lined by non-keratizing squamous epithelium. Thin-walled
capillary vessels, densely packed together with scarce parenchyma is seen in hemangioblastoma.
Perivascular pseudorosettes are often seen in ependymoma and astroblastoma but can be seen
in many neuroendocrine and neurological tumors. Homer wright rosettes can be seen in
medulloblastomas or PNETs. Flexner-Winterstein Rosettes are particularly characteristic of
retinoblastomas but can be seen in pineoblastomas and medulloepitheliomas.
References:
“Müller HL, Merchant TE, Warmuth-Metz M, Martinez-Barbera JP, Puget S. Craniopharyngioma.
Nat Rev Dis Primers. 2019 Nov 7;5(1):75. doi: 10.1038/s41572-019-0125-9. PMID: 31699993
Pubmed Web link
https://pubmed-ncbi-nlm-nih-gov.eresources.mssm.edu/31699993/
Müller HL. Craniopharyngioma. Handb Clin Neurol. 2014;124:235-53. doi:
10.1016/B978-0-444-59602-4.00016-2. PMID: 25248591.
Pubmed Web link
https://pubmed-ncbi-nlm-nih-gov.eresources.mssm.edu/25248591/
Pubmed Web link: https://pubmed-ncbi-nlm-nih-gov.eresources.mssm.edu/8052376/
Torné R, Urra X, Topczeswki TE, Ferrés A, García-García S,
Rodríguez-Hernández A, San Roman L, de Riva N, Enseñat J. Intraoperative
magnetic resonance imaging for cerebral cavernous malformations: When is it maybe worth it? J
Clin Neurosci. 2021 Jul;89:85-90. doi: 10.1016/j.jocn.2021.04.017. Epub 2021 May 6. PMID:
34119300.
Pubmed Web link: https://pubmed-ncbi-nlm-nih-gov.eresources.mssm.edu/34119300/”

How well did you know this?
1
Not at all
2
3
4
5
Perfectly
8
Q

Central pontine myelinolysis is characterized by which of the following?
Answers:
A. Osmotic demyelination
B. Vasogenic edema
C. Blood Brain Barrier disruption and protein extravasation
D. Small linear areas of bleeding
E. Cytotoxic edema

A

Osmotic demyelination

Discussion:
Central pontine myelinolysis (CPM) or extrapontine myelinolysis (EPM) may occur during the rapid
correction of hyponatremia. The blood brain barrier (BBB) is likely disrupted, creating a shift and
disruption of electrolyte imbalance leading to demyelination and destruction of white matter.
Cytotoxic edema is the accumulation of intracellular fluid. This is often seen following an ischemic
event.
Vasogenic edema is the accumulation of extracellular fluid and proteins due disruption of the BBB
and protein extravasation. This edema is typically found surrounding neoplastic or infectious
lesions.
Small linear hemorrhages in the midbrain and upper pons (Duret hemorrhage) are due to
transtentorial herniation.
References:
Norenberg MD. Central pontine myelinolysis: historical and mechanistic considerations. Metab
Brain Dis. Mar 2010;25(1):97-106. doi:10.1007/s11011-010-9175-0
Martin RJ. Central pontine and extrapontine myelinolysis: the osmotic demyelination syndromes. J
Neurol Neurosurg Psychiatry. Sep 2004;75 Suppl 3:iii22-8. doi:10.1136/jnnp.2004.045906

How well did you know this?
1
Not at all
2
3
4
5
Perfectly
9
Q

A 28-year-old man presents with headaches, nausea, and vomiting. A CT scan of the head (Figure A) and MR images of the brain (Figures B and C) are shown. A photomicrograph (H & E stain) of a biopsy specimen is shown (Figure D). Which of the following is the most likely diagnosis?
Answers:
A. Neurocysticercosis
B. Cerebral abscess
C. Neurocytoma
D. Metastasis
E. Colloid cyst

A

Neurocysticercosis

Discussion:
Neurocysticercosis is the most common helminthic infection of the nervous system and a leading
cause of acquired epilepsy worldwide. The disease occurs when humans become intermediate
hosts of the tapeworm Taenia solium by ingesting its eggs from contaminated food or, most often,
directly from a Taenia carrier by the fecal-oral route. Once in the human intestine, Taenia eggs
evolve to oncospheres that cross the intestinal wall and lodge in human tissues - especially the
nervous system - where cysticerci develop. Cysticerci may be in the brain parenchyma,
subarachnoid space, ventricular system, or spinal cord, causing a myriad of pathologic changes
responsible for the clinical pleomorphism of neurocysticercosis. Seizures are the most common
clinical manifestation of the disease, but some patients present with focal deficits, intracranial
hypertension, or cognitive decline. Neuroimaging may demonstrate cystic lesions with the scolex
(head) as an eccentric nodule, as shown above in Figure C. Viable intraparenchymal cysts are
seen as well-defined, rounded cystic structures with liquid contents with signal like CSF and
minimal or no surrounding inflammation. Once the parasite is detected by the immune system of
the host, inflammation is initially noticeable as pericystic edema and contrast enhancement,
followed by the collapse of the cyst into a small annular or nodular lesion that resolves and then
eventually reappears in most cases as a small nodular calcified lesion. Histologically, cysticerci
manifest in the vesicular stage as vesicles with viable organisms. Each viable organism is
composed of a larva containing an invaginated scolex and is surrounded by translucent fluid that is
lined by a thin membranous wall, as seen in Figure D. Inflammatory response is usually induced
by degenerating cysticerci, with predominant lymphocytes and plasma cells, variable edema,
gliosis, fibrosis, and necrosis.
Treatment is usually with albendazole and praziquantel. Surgical intervention may be needed in
cases of obstructive hydrocephalus caused by intraventricular cysts, cysts in the basal cisterns, or
cyst clusters. Hydrocephalus is usually resolved by means of VP shunt or by neuroendoscopic
excision of intraventricular lesions with third ventricular fenestration.
The imaging and pathology do not match that of a cerebral abscess, for which an intraventricular
enhancing nodule would be atypical. Although colloid cysts and neurocytomas are intraventricular
lesions, again the imaging findings with an enhancing nodule and pathology with a larva are
inconsistent with these processes. Intraparenchymal cysticerci may sometimes appear like
metastases, but intraventricular metastases as above would be less likely.
References:
Wallin MT, Kurtzke JF. Neurocysticercosis in the United States: review of an important emerging
infection. Neurology. 2005;63:1559-64.
Pubmed Web link: https://pubmed.ncbi.nlm.nih.gov/15851761/
Garcia HH. Neurocysticercosis. Neurol Clin. 2018;36(4):851-864.
Pubmed Web link: https://pubmed.ncbi.nlm.nih.gov/30366559/
Del Brutto OH. Neurocysticercosis. Handb Clin Neurol. 2014;121:1445-1459.
Pubmed Web link: https://pubmed.ncbi.nlm.nih.gov/24365429/

How well did you know this?
1
Not at all
2
3
4
5
Perfectly
10
Q

A 60-year-old woman is evaluated because of a one-month history of headaches. An MR image is
obtained and a biopsy is done. Photomicrographs (H & E and immunohistochemical stains) are
shown. Which of the following is the most likely diagnosis?
Answers:
A. Cerebral abscess
B. Cerebral metastatic tumor
C. Primary CNS lymphoma
D. Glioblastoma
E. Progressive multifocal leukoencephalopathy

A

Primary CNS lymphoma

Discussion:
Critique:
Primary CNS lymphoma are commonly located in the corpus callosum, basal ganglia, and
periventricular white matter. Histologically, they are characterized by proliferation of discohesive,
diffuse infiltrating atypical lymphoid cells. The majority of these tumors are B cell lymphomas,
which are positive for CD20 and CD79a staining. A smaller percentage are T-cell lymphomas,
which stain positive for CD3.
Glioblastoma can have similar appearance on MRI, although multi-focal GBM is rare.
Histologically, GBM are characterized by hypercellularity with intermingling neoplastic and nonneoplastic elements, and pseudopalisading necrosis. GFAP staining is positive.
Cerebral metastases are characterized histologically by a well demarcated border between tumor
and adjacent reactive gliotic and edematous brain parenchyma. Vascular proliferative changes
may be abundant. Immunostaining is representative of the parental tumor.
Progressive multifocal leukoencephalopathy can appear similar on MRI to Diffuse Large B-Cell
Lymphoma, but histologically they are differentiated by the presence of eosinophilic intranuclear
inclusion bodies.
Cerebral abscesses demonstrate significant necrosis with inflammation on histology.
References:
Reference (1)
Said J. Diffuse aggressive B-cell lymphomas. Adv Anat Pathol. 2009 Jul;16(4):216-35. doi:
10.1097/PAP.0b013e3181a9d5d2. PMID: 19546610.
Pubmed Web link
https://pubmed.ncbi.nlm.nih.gov/19546610
Reference (2)
Sehn LH, Salles G. Diffuse Large B-Cell Lymphoma. N Engl J Med. 2021 Mar 4;384(9):842-858.
doi: 10.1056/NEJMra2027612. PMID: 33657296; PMCID: PMC8377611.
Pubmed Web link
https://pubmed.ncbi.nlm.nih.gov/33657296

How well did you know this?
1
Not at all
2
3
4
5
Perfectly
11
Q

Which of the following diagnostic methods is most appropriate for a patient who has suspected
herpes encephalitis?
Answers:
A. Lumbar puncture with CSF sent for HSV PCR analysis
B. Lumbar puncture with CSF sent for HSV viral culture
C. Serum sent for HSV viral culture
D. Temporal lobe biopsy
E. 24-hour EEG analysis

A

Lumbar puncture with CSF sent for HSV PCR analysis

Discussion:
Herpes simplex virus (HSV) type 1 and 2 are DNA viruses, belonging to the herpes virus family.
These groups of viruses, in addition to primary infection, are known to establish latency in sensory
ganglia and cause recurrent disease manifesting as cutaneous eruptions by reactivation in an
immunocompromised host. These viruses may manifest as commonly recognized cold sores, or
less frequently as herpetic keratitis, presumably related to the latency in trigeminal ganglia. The
pathogenetic association between latent HSV infections and HSV encephalitis is not well
understood. The encephalitis caused by HSV-1 in children and adults is much more circumscribed
and is limited to inferior frontal, medial temporal and adjacent insular cortex, and cingulate gyrus,
while sparing the parieto-occipital lobes and cerebellum. HSV encephalitis in children and adults is
the most frequent nonendemic cause of acute necrotizing encephalitis.
HSV-1 can be easily cultured from the brain of patients, whereas successful isolation from the CSF
is noted only with overwhelming infection. However, the viral genome can be amplified from the
CSF by polymerase chain reaction (PCR) technology. PCR for HSV-1 and HSV-2, which has
supplanted viral cultures and other studies as the test of choice, should be obtained from the CSF
and has high sensitivity (96%) and specificity (99%). HSV serologies are generally not clinically
helpful in the acute setting. Temporal lobe biopsy would not be an appropriate initial diagnostic
step. EEG can demonstrate periodic discharges, focal or generalized slowing, and seizures
including status epilepticus. However, while EEG is recommended as part of the diagnostic
evaluation, there are few studies characterizing the contribution of EEG to diagnosis and prognosis
in these patients.
References:
Love S, Wiley CA. Viral infections. In: Love S, Louis DN, Ellison DW, eds. Greenfield’s
Neuropathology. 8th ed. London: Edward Arnold Ltd; 2008: 1275-1389.
Bradshaw MJ, Venkatesan A. Herpes Simplex Virus-1 Encephalitis in Adults: Pathophysiology,
Diagnosis, and Management. Neurotherapeutics. 2016;13(3):493-508.
Pubmed Web link: https://pubmed.ncbi.nlm.nih.gov/27106239/
Shankar SK, Mahadevan A, Kovoor JM. Neuropathology of viral infections of the central nervous
system. Neuroimaging Clin N Am. 2008;18(1):19-vii.
Pubmed Web link: https://pubmed.ncbi.nlm.nih.gov/18319153/

How well did you know this?
1
Not at all
2
3
4
5
Perfectly
12
Q

An 8-year-old girl is evaluated because of headaches and diplopia. A contrast-enhanced T1-
weighted MR image is shown. Which of the following is the most likely diagnosis?
Answers:
A. Pleomorphic Xanthoastrocytoma
B. Juvenile Pilocytic Astrocytoma
C. Medulloblastoma
D. Ependymoma
E. Hemangioblastoma

A

Juvenile Pilocytic Astrocytoma

Discussion:
The most likely diagnosis is juvenile pilocytic astrocytoma. This is the most commonly occurring
posterior fossa brain tumor in children. There is a strong association with NF1. The most common
location is in the posterior fossa, where there is a cystic lesion with a strongly enhancing mural
nodule. There is avid enhancement of the nodule but not always of the rim. Headaches and
diplopia are common symptoms owing to the elevation of intracranial pressure caused by
blockages of the cerebrospinal fluid pathways.
The most similar diagnosis would be hemangioblastoma, although the mural nodule is often much
smaller. These lesions have a strong association with von Hippel Lindau.
Medulloblastomas are not typically cystic lesions.
Ependymomas often arise from the foramina of Luschka and Magendie and may be central or
peripheral in location.
Although pleomorphic xanthoastrocytoma do have a similar nodular appearance, these lesions are
often supratentorial and located in the temporal lobe, causing temporal lobe epilepsy.
References:
Udaka YT, Packer RJ. Pediatric Brain Tumors. Neurol Clin. 2018 Aug;36(3):533-556.
Pubmed Web link: https://pubmed.ncbi.nlm.nih.gov/30072070/
Kerleroux B, Cottier JP, Janot K, Listrat A, Sirinelli D, Morel B. Posterior fossa tumors in children:
Radiological tips & tricks in the age of genomic tumor classification and advance MR technology. J
Neuroradiol. 2020 Feb;47(1):46-53.
Pubmed Web link: https://pubmed.ncbi.nlm.nih.gov/31541639

How well did you know this?
1
Not at all
2
3
4
5
Perfectly
13
Q

On autopsy of a patient, diffuse spinal cord lesions are found in spinal white matter. These findings
are most consistent with which of the following diagnoses?
Answers:
A. Acute disseminated encephalomyelitis (ADEM)
B. Neuromyelitis optica (Devic disease)
C. Transverse myelitis
D. Multiple sclerosis
E. Amyotrophic lateral sclerosis

A

Multiple sclerosis

Discussion:
Multiple sclerosis in the spine demonstrates multiple lesions on T2 weighted MRI that are typically
shorter than a single vertebral body in length. These plaques often accompany those found on
brain imaging. Identification of demyelinating lesions is important in distinguishing this pathology
from neoplastic or infectious/inflammatory conditions.
NMO lesions are more extensive than MS in the spinal cord, extending >3 vertebral bodies.
ALS does not typically demonstrate any characteristic spinal imaging and is mainly a clinical
diagnosis.
Transverse myelitis is typically diagnosed clinically as an acute process. Nonetheless,
radiographically this typically presents in the thoracic spine as a single lesion.
ADEM typically presents following viral infection or vaccine with multiple lesions.
References:
Filippi M, Rocca MA. MR imaging of multiple sclerosis. Radiology. Jun 2011;259(3):659-81.
doi:10.1148/radiol.11101362
Tillema JM, Pirko I. Neuroradiological evaluation of demyelinating disease. Ther Adv Neurol
Disord. Jul 2013;6(4):249-68. doi:10.1177/1756285613478870

How well did you know this?
1
Not at all
2
3
4
5
Perfectly
14
Q

Immunohistochemical analysis of the perivascular cells in an acute multiple sclerosis lesion is most
likely to show which of the following cell types?
Answers:
A. B cells
B. NK cells
C. T cells
D. Microglia
E. Neutrophils

A

T cells

Discussion:
In the acute inflammatory phase of MS, the T cells are the overwhelming cell type present, much
more so than B cells. Multiple sclerosis (MS) is a demyelinating inflammatory disorder of the
central nervous system (CNS), which involves autoimmune responses to myelin antigens. Studies
in experimental autoimmune encephalomyelitis (EAE), an animal model for MS, have provided
convincing evidence that T cells specific for self-antigens mediate pathology in these diseases.
Microglia are parenchymal. While macrophages can be present in the acute phase in the
perivascular space, they are also present throughout the inflammatory process chronically.
NK cells are part of the innate immune system and historically have not been described to play a
major role in the development of MS, although there is some emerging evidence that NK cells may
play a small role.
Although neutrophils are the most abundant leukocyte, they have not been demonstrated to play a
large role in the inflammatory process of MS.
References:
Dendrou CA, Fugger L, Friese MA. Immunopathology of multiple sclerosis. Nat Rev Immunol. Sep
15 2015;15(9):545-58. doi:10.1038/nri3871
Lassmann H. Multiple Sclerosis Pathology. Cold Spring Harb Perspect Med. Mar 1 2018;8(3).
doi:10.1101/cshperspect.a028936
Mimpen M, Smolders J, Hupperts R, Damoiseaux J. Natural killer cells in multiple sclerosis: A
review. Immunol Lett. Jun 2020;222:1-11. doi:10.1016/j.imlet.2020.02.012

How well did you know this?
1
Not at all
2
3
4
5
Perfectly
15
Q

Which of the following classes of drugs has been shown to induce involution of the lesion in the
pathology specimen shown?
Answers:
A. HMG-CoA reductase inhibitors
B. ACE inhibitors
C. GPIIb/IIIa receptor inhibitors
D. P2Y12 inhibitors
E. Cyclooxygenase inhibitors

A

HMG-CoA reductase inhibitors

Discussion:
Critique:
Carotid stenosis is a pathologic condition caused by progressive atherosclerotic plaque formation
at the carotid artery bifurcation that predisposes patients to developing thromboembolic ischemic
strokes. HMG-CoA reductase inhibitors, also known as statins, are the only group of medications
that have been shown to induce regression of atherosclerotic plaque formation. Anti-platelet
medications, such as aspirin (cyclooxygenase inhibitor), clopidogrel (P2Y12 inhibitor), and
ticagrelor (GIIb/IIIa receptor inhibitor) have demonstrated a decreased risk in developing strokes in
patients with carotid stenosis; however, these medications have not been found to reduce
atherosclerotic plaque burden. ACE inhibitors are a common group of anti-hypertensive
medications and have not been found to reduce atherosclerotic plaque burden.
References:
Reference (1)
Gage BF, van Walraven C, Pearce L, Hart RG, Koudstaal PJ, Boode BS, Petersen P. Selecting
patients with atrial fibrillation for anticoagulation: stroke risk stratification in patients taking aspirin.
Circulation. 2004 Oct 19;110(16):2287-92. doi: 10.1161/01.CIR.0000145172.55640.93. Epub 2004
Oct 11. PMID: 15477396.
Pubmed Web link
https://pubmed.ncbi.nlm.nih.gov/15477396/
Reference (2)
Lip GY, Nieuwlaat R, Pisters R, Lane DA, Crijns HJ. Refining clinical risk stratification for predicting
stroke and thromboembolism in atrial fibrillation using a novel risk factor-based approach: the euro
heart survey on atrial fibrillation. Chest. 2010 Feb;137(2):263-72. doi: 10.1378/chest.09-1584.
Epub 2009 Sep 17. PMID: 19762550.
Pubmed Web link
https://pubmed.ncbi.nlm.nih.gov/19762550/

How well did you know this?
1
Not at all
2
3
4
5
Perfectly
16
Q

A 12-year-old boy is evaluated because of a two-week history of severe headache and vomiting.
MR image shows a posterior fossa mass that is obstructing the fourth ventricle. A biopsy is done
and photomicrographs are shown. Which of the following is the most likely diagnosis?
Answers:
A. Medulloblastoma
B. Ganglioglioma
C. Pilocytic astrocytoma
D. Ependymoma
E. Choroid plexus papilloma

A

Pilocytic astrocytoma

Discussion:
Pilocytic astrocytoma are characterized by piloid astrocytic cells with long, thin, hair-like processes.
There is a biphasic appearance of dense and loose cells which can be seen in the left panel. Also
classically found are Rosenthal fibers, which appear as elongated eosinophilic processes (left
panel), as well as abundant PAS-positive eosinophilic granular bodies (right panel).
Medulloblastomas are an embryonal tumor of the posterior fossa and the most common malignant
brain tumor in children. They are characterized histologically by a sheet-like proliferation of small
blue cells with high nuclear to cytoplasm ration. There is also prominent mitotic activity and
necrosis. Homer Wright rosettes can be present.
Ependymomas are neuroepithelial tumors that occur in the supratentorial brain and posterior
fossa. Histologically, they are characterized by perivascular pseudorosettes. They also have a
sharp demarcation separating tumor from brain.
Choroid plexus papillomas are derived from the choroid plexus epithelium. They are characterized
by papillomas marked by fibrovascular papillary cores lined by stratified epithelial cells.
Gangliogliomas can be found in children but are predominantly in the supratentorial space. They
are characterized by the presence of ganglion and glial cells on histology.
References:
Reference (1)
Udaka YT, Packer RJ. Pediatric Brain Tumors. Neurol Clin. 2018 Aug;36(3):533-556. doi:
10.1016/j.ncl.2018.04.009. PMID: 30072070.
Pubmed Web link
https://www.ncbi.nlm.nih.gov/pubmed/30072070
Reference (2)
Pfister S, Hartmann C, Korshunov A. Histology and molecular pathology of pediatric brain tumors.
J Child Neurol. 2009 Nov;24(11):1375-86. doi: 10.1177/0883073809339213. PMID: 19841426.
Pubmed Web link
https://www.ncbi.nlm.nih.gov/pubmed/19841426

How well did you know this?
1
Not at all
2
3
4
5
Perfectly
17
Q

Which of the following methods of imaging is most likely to identify an ischemic stroke within the
first 12 hours of onset?
Answers:
A. Angiography
B. MRI Diffusion Weighted Imaging (DWI)
C. CT scan without contrast
D. Skull Xray
E. CT angiography

A

MRI Diffusion Weighted Imaging (DWI)

Discussion:
DWI is more sensitive than CT for early detection of ischemic stroke. According to Brazzelli et al.,
the summary estimates for DWI were: sensitivity 0.99 (95% CI 0.23 to 1.00), specificity 0.92 (95%
CI 0.83 to 0.97). The summary estimates for CT were: sensitivity 0.39 (95% CI 0.16 to 0.69),
specificity 1.00 (95% CI 0.94 to 1.00).
The earliest CT sign visible is a hyperdense segment of a vessel, representing direct visualization
of the intravascular thrombus/embolus. However, visualization of loss of grey-white matter
differentiation usually is not visible until 3 hrs after stroke in 75% of patients.
Within minutes of arterial occlusion, DWI demonstrates increased signal and reduced ADC values.
If infarction is incomplete then cortical contrast enhancement may be seen as early as 2 to 4
hours.
CT angiography and angiography may identify thrombus within an intracranial vessel, and may
guide intra-arterial thrombolysis or clot retrieval, but it will not necessarily identify the area of
ischemia itself unless enough time has passed for visualization of loss of grey-white matter
differentiation on CT scan
References:
Burke JF, Sussman JB, Morgenstern LB, Kerber KA. Time to stroke magnetic resonance imaging.
J Stroke Cerebrovasc Dis. 2013 Aug;22(6):784-91. doi:
10.1016/j.jstrokecerebrovasdis.2012.03.012. Epub 2012 Apr 26. PMID: 22541605. https://pubmedncbi-nlm-nih-gov.eresources.mssm.edu/22541605/
Brazzelli M, Sandercock PA, Chappell FM, Celani MG, Righetti E, Arestis N, Wardlaw JM, Deeks
JJ. Magnetic resonance imaging versus computed tomography for detection of acute vascular
lesions in patients presenting with stroke symptoms. Cochrane Database Syst Rev. 2009 Oct
7;(4):CD007424. doi: 10.1002/14651858.CD007424.pub2. PMID: 19821415. https://pubmed-ncbinlm-nih-gov.eresources.mssm.edu/19821415/
Taran, S., Mandell, D.M. & McCredie, V.A. CT Perfusion for the Detection of Delayed Cerebral
Ischemia in the Presence of Neurologic Confounders. Neurocrit Care 33, 317–322 (2020).
https://doi.org/10.1007/s12028-020-01005-2

How well did you know this?
1
Not at all
2
3
4
5
Perfectly
18
Q

The lesion in the axial and coronal MR images shown is most consistent with which of the
following?
Answers:
A. Pituitary Adenoma
B. Vertebral Artery Dissection
C. Craniopharyngioma
D. Carotid Aneurysm
E. Meningioma

A

Carotid Aneurysm

Discussion:
The imaging shows an MRI demonstrating a carotid aneurysm, a rare pathology of the carotid
arteries. The two most predominate groups are atherosclerotic and pseudoaneurysms. Other less
common etiologies include true mycotic aneurysm, trauma, fibromuscular dysplasia, spontaneous
dissection, connective tissue disorders, prior radiation, and congenital defects.
On T1 MRI imaging aneurysms appear dark as the vessels they come from, demonstrating a flow
void, or they may show heterogeneous signal intensity. In thrombosed aneurysms there will be
heterogeneous signal depending on the length of the thrombosis. On T2 MRI imaging an
aneurysm will be hypointense.
The location of this flow void is not in the posterior fossa and thus could not be a vertebral artery
aneurysm. The other answer choices would have signals with differing intensities, not dark as this
flow void.
References:
Kraemer CJK, Zhou W. Carotid Aneurysm Review. Int J Angiol. 2019 Mar;28(1):17-19. doi:
10.1055/s-0039-1677675. Epub 2019 Feb 27. PMID: 30880886; PMCID: PMC6417903.
https://pubmed-ncbi-nlm-nih-gov.eresources.mssm.edu/30880886/
Hahn CD, Nicolle DA, Lownie SP, Drake CG. Giant cavernous carotid aneurysms: clinical
presentation in fifty-seven cases. J Neuroophthalmol. 2000 Dec;20(4):253-8. PMID: 11130752.
https://pubmed-ncbi-nlm-nih-gov.eresources.mssm.edu/11130752/

How well did you know this?
1
Not at all
2
3
4
5
Perfectly
19
Q

A 42-year-old woman is evaluated for cervical myelopathy. T2-weighted and contrast-enhanced T1-weighted MR images are shown. This lesion is most consistent with which of the following diagnoses?
Answers:
A. Cavernous Malformation
B. Hemangioblastoma
C. Schwannoma
D. Spinal Cord Astrocytoma
E. Ependymoma

A

Ependymoma

Discussion:
Cervical Spinal Cord Tumor.jpg” class=”img-fluid” />
The most likely diagnosis is cervical ependymoma. Ependymomas often occur in the cervical spine
from the cells lining the central canal and therefore have a central location with hydromyelia from
obstruction of the spinal fluid, involving a significant portion of the spinal cord.
Spinal cavernous malformations often have a “popcorn” appearance due to mixed T1 and T2
signal from blood of various ages. There may be a rim of low T2 signal due to hemosiderin
deposition. Spinal cord astrocytomas arise from the glial cells of the cord itself, causing a bulbous
appearance with patchy enhancement. Hemangioblastomas are cystic lesions with avidly
enhancing nodules, while this lesion has a rather large soft tissue signal with no intrinsic cyst.
Schwannomas arise from the nerve roots and are inherently not limited to the spinal cord; they
show dumbbell morphology on imaging.
References:
Mariano R, Flanagan EP, Weinshenker BG, Palace J. A practical approach to the diagnosis of
spinal cord lesions. Pract Neurol. 2018 Jun;18(3):187-200.
Pubmed Web link: https://pubmed.ncbi.nlm.nih.gov/29500319/
Beh SC, Greenberg BM, Frohman T, Frohman EM. Transverse myelitis. Neurol Clin. 2013
Feb;31(1):79-138. Pubmed Web link: https://pubmed.ncbi.nlm.nih.gov/23186897/

How well did you know this?
1
Not at all
2
3
4
5
Perfectly
20
Q

An 18-year-old man with an intractable seizure disorder undergoes MR imaging of the brain. T2-
weighted (Figure 1) and fluid attenuated inversion recovery (Figure 2) coronal MR images are
shown. These findings are most consistent with which of the following diagnoses?
Answers:
A. Neurocysticercosis
B. Glioblastoma
C. Pleomorphic Xanthoastrocytoma (PXA)
D. Dysembryoplastic Neuroepithelial Tumor (DNET)
E. Mesial temporal sclerosis

A

Mesial temporal sclerosis

Discussion:
These findings are most consistent with a diagnosis of mesial temporal sclerosis. MRI findings
suggest a decrease in hippocampal/temporal volume with T2 hyperintensity. There is also
abnormal morphology within the mesial temporal structures of the affected side.
Although a potential cause for epilepsy, Dysembryoplastic Neuroepithelial Tumor (DNET)
and Pleomorphic Xanthoastrocytoma (PXA) tend to exhibit distinct radiographic findings. DNET
often are T2 bright with significant peri-lesional vasogenic edema while PXA demonstrate a cystic
appearance with an enhancing mural nodule. Glioblastoma is often avidly T2 bright at the rim and
enhances; there is often a central necrotic core which is non-enhancing and T2 hypointense.
These imaging characteristics are unusual for neurocysticercosis due to the single lesion; also, the
different stages of parasitic growth can show differing imaging findings.
References:
Cendes F, Theodore WH, Brinkmann BH, Sulc V, Cascino GD. Neuroimaging of epilepsy. Handb
Clin Neurol. 2016;136:985-1014.
Pubmed Web link: https://pubmed.ncbi.nlm.nih.gov/27430454/
Mathon B, Bédos Ulvin L, Adam C, Baulac M, Dupont S, Navarro V, Cornu P, Clemenceau S.
Surgical treatment for mesial temporal lobe epilepsy associated with hippocampal sclerosis. Rev
Neurol (Paris). 2015 Mar;171(3):315-25
Pubmed Web link: https://pubmed.ncbi.nlm.nih.gov/25746582/

How well did you know this?
1
Not at all
2
3
4
5
Perfectly
21
Q

The morphologic features in the Grocott-Gomori methenamine silver (GMS) stain shown are most
characteristic of central nervous system infection by which of the following?
Answers:
A. Histoplasma capsulatum
B. Coccidioides immitis
C. Cryptococcal neoformans
D. Blastomyces dermatitidis
E. Candida albicans

A

Candida albicans

Discussion:
Candida organisms appear as mats of yeasts measuring 3 to 5 μm in diameter intermingled with
pseudohyphae (also referred to as filaments). The filaments may show periodic constrictions. The
organisms can be seen with H&E, GMS, and PAS stains. The GMS stain above demonstrates
small yeasts intermingled with pseudohyphae and hyphae. Of note, Aspergillus spp. can look
similar, but have septated hyphae with acute angle branching.
Cryptococci are encapsulated, spherical to oval yeast that measure 5 to 10 μm in diameter and
have narrow-based budding. When testing CSF, India ink can be used as a negative stain to
highlight the capsule.
Histoplasma capsulatum are oval 2- to 4-μm yeast that may show narrow-based buds. With H&E
stain, the basophilic yeast cytoplasm is separated from the surrounding tissue by a clear zone
corresponding to the cell wall. The cell wall is highlighted with GMS and PAS stains. Because the
yeasts are initially ingested by macrophages, they appear to be clustered.
Blastomyces dermatitidis in tissue appears as yeasts that measure 8 to 15 μm in diameter, have
thick refractile cell walls, and may show a single, broad-based bud. The thick refractile cell wall of
this organism gives the appearance of a space between the fungal cell contents and the
surrounding tissue when hematoxylin and eosin (H&E) stain is used. Inside the cell wall, the
multiple nuclei of the yeast stain with hematoxylin. The contour of the yeast is best highlighted by
staining the cell wall with fungal silver stains such as GMS or periodic acid-Schiff (PAS) stain.
Spherules of various sizes (10 to 100 μm) with multiple endospores (2 to 5 μm) are characteristic
of coccidioidomycosis and can be seen with routine H&E staining. The walls of some of the
spherules may appear to be ruptured, and the endospores spill into surrounding tissues. GMS
highlights spherule walls and endospores.
References:
Burger P, Scheithauer B, eds. Diagnostic Pathology: Neuropathology. 1st ed. Philadelphia, PA:
Lippincott Williams & Wilkins; 2012.
Nathan CL, Emmert BE, Nelson E, Berger JR. CNS fungal infections: A review. J Neurol Sci.
2021;422:117325.
Pubmed Web link: https://pubmed.ncbi.nlm.nih.gov/33516057/
Guarner J, Brandt ME. Histopathologic diagnosis of fungal infections in the 21st century. Clin
Microbiol Rev. 2011;24(2):247-280.
Pubmed Web link: https://www.ncbi.nlm.nih.gov/pmc/articles/PMC3122495/

How well did you know this?
1
Not at all
2
3
4
5
Perfectly
22
Q

A 67-year-old woman presents with a ring-enhancing mass in her left thalamus. A biopsy of the
lesion is shown. Which of the following is the most appropriate next step in management?
Answers:
A. Start Amphotericin
B. Start vancomycin and ceftriaxone
C. Attempt to resect the thalamic mass
D. Observation
E. Start pyrimethamine and sulfadiazine

A

Start pyrimethamine and sulfadiazine

Discussion:
The image above demonstrates Toxoplasma gondii parasites within a cyst. T. gondii is an
opportunistic parasite that can infect many domestic and wild animals with subsequent
transmission to humans. People become infected primarily through consuming raw or improperly
cooked meat (particularly lamb and pork) containing infectious tissue cysts or via ingestion of
sporulated oocysts in vegetables, fruits, or water contaminated with feline feces. At the initial
infection site in the intestine, T. gondii infects various immune cells and use them to migrate to and
infiltrate the brain, where it employs various strategies to overcome the blood-brain barrier. In most
individuals with competent immune responses, primary infection is asymptomatic or may produce
a mild, flu-like illness, and the parasite eventually lies dormant within a tissue cyst. However, in
less than 10% of infections, a mononucleosis-like syndrome with headache, malaise, fever,
cervical lymphadenopathy, and fatigue may occur. Primary T. gondii infection can also cause
ocular disease,, and in pregnant women, can lead to fetal death or brain damage in congenitally
infected children.
Cerebral toxoplasmosis most commonly presents as one or, more commonly, multiple brain
abscesses with a predilection for the deep gray matter structures or the junction between cortical
gray and white matter. CT and MRI can reveal these ring-enhancing lesions, and an eccentric
“target sign” enhancement on MRI has been described (Figure 1). Common neurological
abnormalities in immunocompromised patients (e.g., with HIV) include subacute onset of
headache, hemiparesis, cranial nerve palsies, ataxia, change in level of consciousness, or
seizures. Due to involvement of the basal ganglia, abnormal movements, including chorea,
ballism, and rigidity may also be observed.
The Sabin-Feldman dye test remains the “gold standard” for serological detection of antiToxoplasma IgG and IgM antibodies. However, this method requires using live tachyzoites, which
is not feasible in most laboratories. An enzyme-linked immunosorbent assay (ELISA) is commonly
used to detect specific IgG and IgM antibodies against T. gondii. Mild mononuclear pleocytosis and
elevated protein can be detected in the CSF of patients with toxoplasmosis. CNS toxoplasmosis
can lead to granulomatous reactions with gliosis and microglial nodules and necrotizing
encephalitis. The detection of tachyzoites either alone or together with tissue cysts is diagnostic,
as demonstrated in the image above. In latent infection, slow-growing bradyzoites are encased in
cysts, and multiple basophilic dot-like parasites are seen within.
Treatment is usually with pyrimethamine combined with sulfadiazine or clindamycin and
leucovorin. An alternative treatment is with trimethoprim-sulfamethoxazole. Observation and
surgical resection would not be appropriate measures. Vancomycin and ceftriaxone would not be
the appropriate antibiotics for toxoplasmosis. Amphotericin is an antifungal medication.
References:
Dedicoat M, Livesley N. Management of toxoplasmic encephalitis in HIV-infected adults (with an
emphasis on resource-poor settings). Cochrane Database Syst Rev. 2006 Jul 19;(3):CD005420.
Pubmed Web link: https://pubmed.ncbi.nlm.nih.gov/16856096/
Elsheikha HM, Marra CM, Zhu XQ. Epidemiology, Pathophysiology, Diagnosis, and Management
of Cerebral Toxoplasmosis. Clin Microbiol Rev. 2020;34(1):e00115-19.
Pubmed Web link: https://pubmed.ncbi.nlm.nih.gov/33239310/

How well did you know this?
1
Not at all
2
3
4
5
Perfectly
23
Q

The sagittal non-contrast T1-weighted MR image (Figure A) and the axial T2-weighted MR image
(Figure B) shown are from a patient with sudden onset of diplopia and mild right hemiparesis.
Which of the following is the most likely diagnosis?
Answers:
A. Glioblastoma
B. Dermoid Cyst
C. Epidermoid Cyst
D. Tectal Glioma
E. Cavernous Malformation

A

Cavernous Malformation

Discussion:
The most likely diagnosis is a cavernous malformation. Such lesions can occur anywhere within
the neuroaxis and often have a “popcorn” appearance at the rim due to hemosiderin-related signal
loss. T1 and T2 demonstrate variable intensity depending on the age of the blood products
surrounding the lesion after bleeding.
Dermoid cysts are often extra-axial and show T1 hyperintensity due to their cholesterol content.
Epidermoid cysts are also extra-axial, filling the CSF space and iso- to hyperintense to CSF in both
the T1 and T2 sequences. This lesion is in the midbrain and not in the collicular plate, as would be
needed to be a tectal glioma. Tectal glioma lesions are often T1 iso- to hypointense to grey matter
and T2 bright to grey matter. Glioblastoma often demonstrates areas of necrosis with
heterogenous T1 signal at the necrotic core, as well as ring-like enhancement.
References:
Rapalino O, Smirniotopoulos JG. Extra-axial brain tumors. Handb Clin Neurol. 2016;135:275-291.
Pubmed Web link: https://pubmed.ncbi.nlm.nih.gov/27432671/
Petr O, Lanzino G. Brainstem cavernous malformations. J Neurosurg Sci. 2015 Sep;59(3):271-82.
Epub 2015.
Pubmed Web link: https://pubmed.ncbi.nlm.nih.gov/25943871/

How well did you know this?
1
Not at all
2
3
4
5
Perfectly
24
Q

A 43-year-old man receiving chemotherapy for acute myelogenous leukemia develops fever and
generalized tonic-clonic seizures. An MR image of the brain shows multiple ring-enhancing
cerebral lesions. The largest lesion is biopsied and a photomicrograph is shown. Which of the
following is the most likely causal organism?
Answers:
A. Cryptococcus neoformans
B. Blastomyces dermatitidis
C. Candida albicans
D. Coccidioides immitis
E. Histoplasma capsulatum

A

Candida albicans

Discussion:
Candida organisms appear as mats of yeasts measuring 3 to 5 μm in diameter intermingled with
pseudohyphae (also referred to as filaments). The filaments may show periodic constrictions. The
organisms can be seen with H&E, GMS, and PAS stains. The H&E stain above demonstrates
pseudohyphae and hyphae with small budding yeast cells, in an immunocompromised host. Of
note, Aspergillus spp. can look similar, but have septated hyphae with acute angle branching.
In Cryptococcus neoformans, pleomorphic yeast-like cells and formation of narrow-based buds are
typical. In Coccidioides immitis, spherules containing endospores are the typical structure. In
Histoplasma capsulatum, intracellular budding yeast cells are typical. With Blastomyces
dermatitidis infections, thick-walled yeast-like cells and single broad-based buds are typical.
References:
Nathan CL, Emmert BE, Nelson E, Berger JR. CNS fungal infections: A review. J Neurol Sci.
2021;422:117325.
Pubmed Web link: https://pubmed.ncbi.nlm.nih.gov/33516057/
Guarner J, Brandt ME. Histopathologic diagnosis of fungal infections in the 21st century. Clin
Microbiol Rev. 2011;24(2):247-280.
Pubmed Web link: https://www.ncbi.nlm.nih.gov/pmc/articles/PMC3122495

How well did you know this?
1
Not at all
2
3
4
5
Perfectly
25
Q

A 76-year-old man is evaluated for a three-month history of bilateral leg weakness and pain, which is worse on the left. Muscle strength of the left dorsiflexors is 4/5 and sensation to touch is mildly decreased in the left lateral calf and foot. Based on the MR image and photomicrograph (H & E
stain) shown, which of the following is the most likely diagnosis?
Answers:
A. Synovial cyst
B. Herniated disc
C. Osteoid osteoma
D. Arachnoid cyst
E. Schwannoma

A

Synovial cyst

Discussion:
Based on the clinical history, imaging, and histology, the diagnosis is that of a synovial cyst.
Synovial cysts are benign herniations of the synovium of a facet joint. They occur due to
degenerative changes, and are asymptomatic except for when they impinge on a nerve root,
where they can cause radiculopathy, numbness, and weakness. T2 weighted MRI will be high
signal (bright), indicative of fluid within the cyst. Pathology shows fluid filled synovium with an
epithelial lining and myxoid degeneration. Disc herniations should be part of the differential of a
synovial cyst, but are hypointense on T2 weighted imaging. Histopathology would show only
connective tissue without cystic changes. Osteoid osteomas are primary bone lesions, do not
invade into the spinal canal, are hypointense on T2-weighted imaging, and will not have cystic
changes on histopathology. Schwannomas are nerve-sheath tumors, and can be hyperintense on
T2 weighted MRI. However, histopathology would not show cysts but cellular areas (Antoni A),
nuclear palisading (Verocay bodies), and hypocellular areas (Antoni B). Arachnoid cysts are
benign outpouching of the arachnoid and are normally incidental on imaging, rarely causing
symptoms. They are similar to synovial cysts on MRI, but histopathology would not show an
epithelial lining of these cysts.
References:
Berger PC, Scheithauer BW, Vogel FS. Spine and epidural space. Surgical Pathology of the
Nervous System and Its Coverings. In: Berger PC, Scheithauer BW, Vogel FS, eds. Philadelphia:
Elsevier; 2002: 499-526.
Chebib I, Chang CY, Schwab JH, Kerr DA, Deshpande V, Nielsen GP. Histopathology of synovial
cysts of the spine. Histopathology. 2018 May;72(6):923-929. doi: 10.1111/his.13465. Epub 2018
Mar 5. PMID: 29314220.
Kwee RM, Kwee TC. Imaging of facet joint diseases. Clin Imaging. 2021 Dec;80:167-179. doi:
10.1016/j.clinimag.2021.07.005. Epub 2021 Jul 26. PMID: 34333352.

26
Q

The characteristic pathologic finding associated with progressive multifocal leukoencephalopathy
consists of
Answers:
A. Foamy Macrophages
B. Alzheimer type II astrocytes
C. Rosenthal fibers
D. Gemistocytic astrocytes with eccentric nuclei
E. Oligodendroglia-like cells with viral inclusions in their enlarged nuclei

A

Oligodendroglia-like cells with viral inclusions in their enlarged nuclei

Discussion:
Large abnormal oligodendrocytes with viral inclusions are the classic histopathological
representation of progressive multifocal leukoencephalopathy (PML). PML is a devastating
progressive disease of the white matter tracts found in immunocompromised patients with the JC
virus.
Alzheimer type II astrocytes are associated with hepatic encephalopathy.
Gemistocytic astrocytes with eccentric nuclei are reactive astrocytes associated with secondary
astrocytosis following stroke, degenerative diseases, etc.
Rosenthal fibers eosinophilic masses associated with Alexander disease, pilocytic astrocytomas,
and reactive astrocytosis.
Foamy macrophages due to the uptake of lipoproteins are non-specific and may be seen with PML
as well as Multiple Sclerosis.
References:
“Comprehensive Neurosurgery Board Review” Citow JS, MacDonald RL, Puffer R, Khalid S, Carter
B (eds) 3rd Ed (2020) Thieme Publishing Group, page 182.
Shishido-Hara Y. Progressive multifocal leukoencephalopathy: Dot-shaped inclusions and virushost interactions. Neuropathology. Oct 2015;35(5):487-96.
Pubmed Web link: doi:10.1111/neup.12203

27
Q

A gross specimen is shown. Which of the following is the most likely diagnosis?
Answers:
A. Wernicke Encephalopathy
B. Alzheimer disease
C. Pick Disease
D. Parkinson disease
E. Huntington disease

A

Huntington disease

Discussion:
This coronally cut gross specimen of the brain shows degeneration of the bilateral caudate nuclei,
which are present just lateral to the lateral ventricles. Selective degeneration of the caudate
nucleus is a hallmark of Huntington’s disease. On gross pathology, Wernicke encephalopathy is
defined by hemorrhage of the mamillary bodies. Pick’s disease, or frontotemporal dementia, has
atrophy and degeneration of the bilateral frontal and temporal lobes. Alzheimer’s disease would
show cortical degeneration in all areas of the brain. Parkinson’s disease shows degeneration of the
substantia nigra pars compacta.
References:
Ross CA, Tabrizi SJ. Huntington’s disease: from molecular pathogenesis to clinical treatment.
Lancet Neurol. 2011 Jan;10(1):83-98.
Jimenez-Sanchez M, Licitra F, Underwood BR, Rubinsztein DC. Huntington’s Disease:
Mechanisms of Pathogenesis and Therapeutic Strategies. Cold Spring Harb Perspect Med. 2017
Jul 5;7(7):a024240. doi: 10.1101/cshperspect.a024240. PMID: 27940602;

28
Q

A 15-year-old boy undergoes MR imaging for a one-month history of headache, blurred vision,
nausea, and vomiting. A sagittal MR image is shown. In addition to ventriculomegaly, which of the
following findings is demonstrated?
Answers:
A. Pineal Region Tumor
B. Aqueductal Stenosis
C. Choroid Plexus Papilloma
D. Chiari Malformation
E. Tectal Glioma

A

Aqueductal Stenosis

Discussion:
In addition to ventriculomegaly, the most likely finding is aqueductal stenosis. The imaging findings
show a web over the aqueduct of Sylvius, causing triventicular hydrocephalus. Symptoms of
increased intracranial pressure may be present, including headache, vomiting, and decreased
conscious state.
There is no mass-like lesion in the collicular plate, pineal gland, or within the ventricles to suggest
Tectal Glioma, Pinal Region Tumor or Choroid Plexus Papilloma, respectively. Moreover, the
presence of a cisterna magna and lack of tonsillar descent rules out Chiari Malformation.
References:
Cinalli G, Spennato P, Nastro A, Aliberti F, Trischitta V, Ruggiero C, Mirone G, Cianciulli E.
Hydrocephalus in aqueductal stenosis. Childs Nerv Syst. 2011 Oct;27(10):1621-42.
Pubmed Web link: https://pubmed.ncbi.nlm.nih.gov/21928028/
Kahle KT, Kulkarni AV, Limbrick DD Jr, Warf BC. Hydrocephalus in children. Lancet. 2016 Feb
20;387(10020):788-99. Pubmed Web link: https://pubmed.ncbi.nlm.nih.gov/26256071/

29
Q

The histologic specimen from the ventral horn of the spinal cord shown was stained with
immunoperoxidase-ubiquitin. Which of the following is the most likely diagnosis?
Answers:
A. Amyotrophic lateral sclerosis
B. Ependymoma
C. Subacute combined degeneration
D. Multiple sclerosis
E. Poliomyelitis

A

Amyotrophic lateral sclerosis

Discussion:
The histopathology slide shows staining within a large cell from the ventral horn, indicating that this
is likely a ventral motor neuron. Immunoperoxidase-ubiquitin staining stains for the ubiquitin
protein; ubiquitin is a protein that targets other proteins for degradation, and accumulates
abnormally in neurodegenerative disorders, including idiopathic amyotrophic lateral sclerosis.
Thus, the combination of a large ventral horn cell with ubiquitin staining confirms the diagnosis of
amyotrophic lateral sclerosis and excludes the other diagnoses. Subacute combined degeneration
occurs due to vitamin B12 deficiency; it involves degeneration of the dorsal columns (white matter
lesion). Poliomyelitis does involve anterior horn cells, but is not classified by intracellular
accumulations rather by necrosis of these cells. Ependymomas are primary tumors of the
ependymal lining within the central canal and does not stain for ubiquitin. Multiple sclerosis
normally produces white matter lesions and is not associated with intracellular staining.
References:
National Institute of Neurological Disorders and Stroke. Amyotrophic Lateral Sclerosis (ALS)
Information Page. NINDS Web site. Available at: www.ninds.gov/disorders/amyotrophic lateral
sclerosis/detal_ALS.htm. Updated October 21, 2013.
Saberi S, Stauffer JE, Schulte DJ, Ravits J. Neuropathology of Amyotrophic Lateral Sclerosis and
Its Variants. Neurol Clin. 2015;33(4):855-876. doi:10.1016/j.ncl.2015.07.012
Leigh P, Anderton B, Dodson A, Gallo J-M, Swash M, Power D. Ubiquitin deposits in anterior horn
cells in motor neurone disease. Neuroscience letters. 1988;93:197–203

30
Q

A 27-year-old man undergoes resection of an intraventricular mass. A biopsy of the mass is done, and the tissue stains positively for synaptophysin and neuron-specific enolase. A photomicrograph (H&E stain) is shown. Which of the following is the most likely diagnosis?
Answers:
A. Subependymal giant cell astrocytoma
B. Ependymoma
C. Choroid plexus papilloma
D. Oligodendroglioma
E. Central neurocytoma

A

Central neurocytoma

Discussion:
Central neurocytomas are non-glial tumors with a peak incidence between the third and fifth
decades. They are classically located in the lateral ventricles near the foramen of Monroe.
Histologically, they are characterized by proliferating round cells with scant cytoplasm against a
fibrillary background. The nuclei have a finely speckled chromatin pattern. Immunostaining is
positive for synaptophysin, neuronal nuclear antigen (NeuN) and neuron specific enolase (NSE),
although NSE is not specific for central neurocytomas. They also are negative for GFAP, which
differentiates them from oligodendrogliomas which can have a similar histological appearance.
Ependymomas can demonstrate positivity of GFAP, S-100, EMA and CD99. Histologically, they are
characterized by perivascular pseudorosettes.
Subependymal giant cell astrocytomas are characterized by positive GFAP and S-100 staining.
Histologically, they appear as spindled and epithelioid cells, often with perivascular
pseudorosettes.
Choroid plexus papillomas are characterized by positive staining for vimentin, keratin, and S-100
protein. Histologically, they appear as fibrovascular papillary cores lined by stratified epithelial
cells.
Oligodendrogliomas are characterized by positive GFAP staining and a ‘fried-egg’ appearance on
histology.
References:
Reference (1)
Patel DM, Schmidt RF, Liu JK. Update on the diagnosis, pathogenesis, and treatment strategies
for central neurocytoma. J Clin Neurosci. 2013 Sep;20(9):1193-9. doi: 10.1016/j.jocn.2013.01.001.
Epub 2013 Jun 27. PMID: 23810386.
Pubmed Web link
https://www.ncbi.nlm.nih.gov/pubmed/23810386
Reference (2)
Soylemezoglu F, Onder S, Tezel GG, Berker M. Neuronal nuclear antigen (NeuN): a new tool in
the diagnosis of central neurocytoma. Pathol Res Pract. 2003;199(7):463-8. doi:
10.1078/0344-0338-00446. PMID: 14521262.
11.Pubmed Web link
https://www.ncbi.nlm.nih.gov/pubmed/1451262

31
Q

In the histologic slide from the hippocampus (stained with modified Bielschowsky silver stain)
shown, the pathologic cells contain which of the following?
Answers:
A. Neurofibrillary Tangles
B. Pick bodies
C. Negri bodies
D. Lewy bodies
E. Hirano bodies

A

Neurofibrillary Tangles

Discussion:
The Bielschowsky silver stain is used to detect nerves and nerve fibers, especially in the workup of
neurodegenerative disorders. Alzheimer disease is known to affect the hippocampus and has
intracellular inclusions, neurofibrillary tangles and hirano bodies. Neurofibrillary tangles are
elongated intracellular inclusions containing tau protein and ubiquitin. Hirano bodies are also
intracellular inclusions of the protein actin, are found in the hippocampus in Alzheimer disease, but
are rod-shaped. Pick bodies are found in Pick disease, or frontotemporal dementia. They are
spherical cytoplasmic inclusions that have tau protein and ubiquitin. Pick disease is known to affect
the hippocampus, consistent with the location of this lesion. Parkinson’s disease has Lewy bodies,
which are also spherical, but are associated with a halo and are not normally found in the
hippocampus. Rabies produces intracellular Negri bodies within the hippocampus and cerebellum,
consisting of viral proteins, but they are much smaller than Pick bodies. Amyotrophic lateral
sclerosis produces spherical Bunina bodies, consisting of ubiquitin, but they are found in ventral
horn cells.
References:
Brion JP. Neurofibrillary tangles and Alzheimer’s disease. Eur Neurol. 1998 Oct;40(3):130-40. doi:
10.1159/000007969. PMID: 9748670.
DeTure, M.A., Dickson, D.W. The neuropathological diagnosis of Alzheimer’s disease. Mol
Neurodegeneration 14, 32 (2019). https://doi.org/10.1186/s13024-019-0333-5

32
Q

To prevent the pathology in the image shown, which of the following should be administered first?
Answers:
A. Niacin (B3)
B. Thiamine (B1)
C. Folate (B9)
D. Pyridoxine (B6)
E. Cobalamin (B12)

A

Thiamine (B1)

Discussion:
This gross specimen shows bilateral hemorrhage of the mammillary bodies, which is the hallmark
of Wernicke’s encephalopathy caused by Thiamine (B1) deficiency. Wernicke’s encephalopathy
presents clinically with nystagmus, conjugate gaze palsies, ataxia, and confusion, and is a
reversible condition. If left untreated, it will progress to Korsakoff Psychosis, an irreversible
condition defined by retro- and anterograde amnesia and confabulation. Folate deficiency is
associated with neural tube defects. Cobalamin deficiency is associated with subacute combined
degeneration, which can occur in the setting of pernicious anemia. Pyridoxine deficiency can occur
with chronic administration of isoniazid. Niacin deficiency, or pellagra, is associated with dermatitis,
diarrhea, and dementia.
References:
Sullivan EV, Fama R. Wernicke’s encephalopathy and Korsakoff’s syndrome revisited. Neuropsych
Rev. 2012 June;22(2):69-71.
Nishimoto A, Usery J, Winton JC, Twilla J. High-dose Parenteral Thiamine in Treatment of
Wernicke’s Encephalopathy: Case Series and Review of the Literature. In Vivo

33
Q

A 60-year-old man is evaluated because of gradual-onset visual loss, decreased libido, and headaches. MR imaging of the brain was obtained by his primary care physician, and a representative T1-weighted sagittal MR image with gadolinium is shown. Based on the history and imaging characteristics of this lesion, elevated levels of which of the following can be expected in this patient?
Answers:
A. Prolactin
B. Adrenocorticotropic Hormone
C. Non-Functional Adenoma
D. Thyroid Stimulating Hormone
E. Growth Hormone

A

Prolactin

Discussion:
The most likely diagnosis is a pituitary macroadenoma. The nature of the symptoms suggests that
this is most likely a Prolactinoma, releasing the hormone prolactin. The large size of the lesion is
likely to cause bitemporal hemianopsia and headaches from elevated intracranial pressure.
Excess production of the hormone will result in a decrease in libido.
An increase in adrenocorticotropin hormone will cause Cushing’s syndrome, while a growth
hormone producing tumor will cause Acromegaly. Thyroid stimulating hormone excess will cause a
decrease in T3 and Free T4, causing symptoms of hypothyroidism. Given the hormonal sideeffects, this is unlikely a non-functional macroadenoma.
References:
Chapman PR, Singhal A, Gaddamanugu S, Prattipati V. Neuroimaging of the Pituitary Gland:
Practical Anatomy and Pathology. Radiol Clin North Am. 2020 Nov;58(6):1115-1133.
Pubmed Web link: https://pubmed.ncbi.nlm.nih.gov/33040852/
Mazumdar A. Imaging of the pituitary and sella turcica. Expert Rev Anticancer Ther. 2006 Sep;6
Suppl 9:S15-22.
Pubmed Web link: https://pubmed.ncbi.nlm.nih.gov/17004852/

34
Q

Spinal vacuolar myelopathy, leukoencephalopathy, and subacute encephalitis are the most common neuropathologic findings in patients infected with which of the following types of virus?
Answers:
A. Human Immunodeficiency Virus
B. Measles virus
C. Varicella Zoster Virus
D. Epstein-Barr Virus
E. Herpes Simplex Virus

A

Human Immunodeficiency Virus

Discussion:
About 70% of HIV patients develop neurologic complications. These originate either in the central
or the peripheral nervous system. These neurologic disorders are divided into primary and
secondary disorders. The primary disorders result from the direct effects of the virus and include
HIV-associated neurocognitive disorder (HAND), HIV-associated vacuolar myelopathy, and distal
symmetric polyneuropathy. Secondary disorders result from marked immunosuppression and
include opportunistic infections and primary CNS lymphoma.
HIV-associated vacuolar myelopathy usually occurs late in the disease and is characterized by the
subacute progression of spastic paraparesis, bladder dysfunction and sensory loss (primarily
proprioception). The disease is caused by degeneration of the posterior and lateral columns of the
spine. Histologic findings include spongiform change with vacuolization of myelin sheaths, lipidladen macrophages and microglia, and multinucleated giant cells.
Patients with HIV infection may be more susceptible to progressive multifocal
leukoencephalopathy (PML) and cytomegalovirus (CMV) infection. PML results from reactivation of
latent JC virus infection in the setting of immunosuppression. In patients infected with HIV, this is
seen most often with CD4+ cell count less than 200 cells/mm. This may present with visual
dysfunction, neuropsychological changes, focal motor or sensory impairment, ataxia or other gait
disturbance, and speech difficulties. MRI demonstrates asymmetric, non-enhancing T2/FLAIR
hyperintense lesions most commonly involving the juxtacortical white matter within parieto-occipital
regions and the middle cerebellar peduncle. Restricted diffusion may be seen at the leading edge
of expansive lesions and has been attributed to swelling secondary to oligodendrocyte and
astrocyte death.
HIV subacute encephalitis refers to cognitive impairment resulting from productive cerebral
infection, usually occurring late in the disease. Subacute encephalitis may occur anywhere within
the nervous system, but there is a distinct predilection for the subcortical white matter and basal
ganglia. It is characterized by demyelination, gliosis of the gray and white matter, focal necrosis,
microglial nodules, atypical oligodendrocyte nuclei, and multinucleated cells.
Herpes encephalitis most often presents with altered mental status, fever, and MRI findings of
increased T2 signal intensity in the frontotemporal region. The neuropathological picture of herpes
encephalitis is characteristic, consisting of an acute necrotizing encephalitis that almost always
localizes, often asymmetrically, to the orbitofrontal and temporal lobes with involvement of the
cingulate and insular cortex.
Epstein-Barr Virus was first identified in cultures of Burkitt’s lymphoma. CNS involvement is rare,
with features of mild nonspecific meningoencephalitis.
Varicella Zoster causes diffuse exanthematous cutaneous lesions of chickenpox in pediatric
patients and focal dermatomal shingles in elderly patients. Infection of the CNS is rather
uncommon and can manifest as aseptic meningitis, transverse or ascending myelitis,
postinfectious encephalitis, or Guillain-Barre syndrome in adults.
The measles virus belongs to the paramyxovirus family and can cause subacute sclerosing
panencephalitis (SSPE). SSPE is a disease of patients who have no immunologic impairment,
usually manifesting a few years after the initial measles infection. In children and teenagers, SSPE
progresses from initial symptoms of intellectual impairment and behavioral abnormalities to motor
impairment, myoclonus, ataxia, and spasticity to final stage of stupor, autonomic impairment, and
coma. Neuroimaging reveals white matter changes. Histological examination reveals
leptomeningeal, parenchymal, and perivascular inflammation. Neuronal loss, reactive astrocytosis,
large areas of white matter demyelination, and focal microglial responses are seen. Intranuclear
inclusions are seen in neurons and oligodendroglia.
References:
Grill MF. Neurologic Complications of Human Immunodeficiency Virus. Continuum (Minneap Minn).
2021;27(4):963-991.
Pubmed Web link: https://pubmed.ncbi.nlm.nih.gov/34623100/
Bilgrami M, O’Keefe P. Neurologic diseases in HIV-infected patients. Handb Clin Neurol.
2014;121:1321-1344.
Pubmed Web link: https://pubmed.ncbi.nlm.nih.gov/24365422/
Shankar SK, Mahadevan A, Kovoor JM. Neuropathology of viral infections of the central nervous
system. Neuroimaging Clin N Am. 2008;18(1):19-vii.
Pubmed Web link: https://pubmed.ncbi.nlm.nih.gov/18319153/

35
Q

A 19-year-old woman with epilepsy and the MR image shown undergoes surgical resection of the epileptogenic focus. Which of the following histopathologic features will most likely be noted by the neuropathologist?
Answers:
A. Rosenthal fibers
B. Pseudorosettes
C. Homer Wright Rosettes
D. Hippocampal sclerosis
E. Necrosis

A

Hippocampal sclerosis

Discussion:
The MRI shows right mesial temporal lobe sclerosis, which is the most common cause of temporal
lobe epilepsy. Histopathology will show hippocampal sclerosis, which is pathologically defined as
having neuronal loss and glial proliferation. Rosenthal fibers are associated with pilocystic
astrocytomas, which are normally located in the posterior fossa in children and peri-ventricular in
adults. Necrosis seen on pathology is associated with glioblastomas. Pseudorosettes are seen in
ependymomas, which are mass lesions originating from the ventricle. Homer Wright Rosettes are
associated with medulloblastomas and neuroblastomas.
References:
Blümcke I. Neuropathology of focal epilepsies: a critical review. Epilepsy Behav. 2009
May;15(1):34-9.2.
Blümcke I, Pauli E, Clusmann H, et al. A new clinico-pathological classification system for mesial
temporal sclerosis. Acta Neuropathol. 2007 Mar;113(3):235–44.

36
Q

Which of the following is most likely to be observed on histology of pilocytic astrocytoma?
Answers:
A. Eosinophilic granular bodies
B. Lipid laden stromal cells
C. Homer-Wright Rosettes
D. ‘Fried-egg’ appearance
E. Perivascular pseudorosettes

A

Eosinophilic granular bodies

Discussion:
Pilocytic astrocytoma are characterized histologically by a biphasic growth pattern of dense and
loose cells. There is also the presence of eosinophilic granular bodies and Rosenthal fibers.
Perivascular pseudorosettes are commonly associated with ependymomas and subependymal
giant cell astrocytomas.
Homer Wright Rosettes can be found in medulloblastomas.
Lipid laden stromal cells are seen in hemangioblastomas.
Clear perinuclear halos resulting in a ‘fried-egg’ appearance is commonly seen in
oligodendrogliomas.
References:
Reference (1)
Cyrine S, Sonia Z, Mounir T, Badderedine S, Kalthoum T, Hedi K, Moncef M. Pilocytic astrocytoma:
a retrospective study of 32 cases. Clin Neurol Neurosurg. 2013 Aug;115(8):1220-5. doi:
10.1016/j.clineuro.2012.11.009. Epub 2012 Dec 21. PMID: 23265563.
Pubmed Web link
https://www.ncbi.nlm.nih.gov/pubmed/23265563
Reference (2)
Bornhorst M, Frappaz D, Packer RJ. Pilocytic astrocytomas. Handb Clin Neurol. 2016;134:329-44.
doi: 10.1016/B978-0-12-802997-8.00020-7. PMID: 26948364.
Pubmed Web link
https://www.ncbi.nlm.nih.gov/pubmed/26948364

37
Q

Lack of which of the following differentiates the lesion in the image shown from a normal cerebral artery?
Answers:
A. Tunica externa
B. Tunica intima
C. Internal elastic lamina
D. External elastic lamina
E. Tunica media

A

Internal elastic lamina

Discussion:
Critique: The image in Figure 1 depicts an intracranial saccular (“berry”) aneurysm. Histologically,
intracranial saccular aneurysms are characterized by disruption of the internal elastic lamina within the artery wall. Classically, it was taught that the lack of the internal elastic lamina was a congenital defect. However, studies indicate that intracranial saccular aneurysms are the result of inflammatory changes caused by recurrent hemodynamic stress, which is most prominent at the branching points of the cerebral vasculature. This stress, especially when coupled with
environmental (ie. cigarette smoking, hypertension, atherosclerosis, etc.) and/or genetic factors
initiates several different inflammatory pathways. This inflammation ultimately results in weakening
of the extracellular matrix and disruption of the internal elastic lamina of the vessel wall. The
persistent hemodynamic stress then results in an outpouching of the weakened wall and the
formation of a saccular aneurysm. *Image used with permission via Creative Commons Attribution 3.0 (Original source:
https://en.wikipedia.org/wiki/Tunica_intima#/media/File:Blausen_0055_ArteryWallStructure.png)
References:
Reference (1)
Yong-Zhong G, van Alphen HA. Pathogenesis and histopathology of saccular aneurysms: review
of the literature. Neurol Res. 1990 Dec;12(4):249-55. doi: 10.1080/01616412.1990.11739952.
PMID: 1982169.
Pubmed Web link
https://pubmed.ncbi.nlm.nih.gov/1982169/
Reference (2)
Chalouhi N, Hoh BL, Hasan D. Review of cerebral aneurysm formation, growth, and rupture.
Stroke. 2013 Dec;44(12):3613-22. doi: 10.1161/STROKEAHA.113.002390. Epub 2013 Oct 15.
PMID: 24130141.
Pubmed Web link
https://pubmed.ncbi.nlm.nih.gov/24130141/

38
Q

The lesion seen on these MR images (T1-weighted sagittal with gadolinium and T2-weighted coronal) is resected, and histopathology shows a single-layer cuboidal epithelium. Which of the following is the most likely diagnosis?
Answers:
A. Rathke’s Cleft Cyst
B. Lymphocytic Hypophysitis
C. Pituitary Adenoma
D. Craniopharyngioma
E. Empty Sella Syndrome

A

Rathke’s Cleft Cyst

Discussion:
The most likely diagnosis of the patient whose MRI is shown is a Rathke’s Cleft Cyst. These are
benign cystic like lesions of the Rathke’s pouch of the pituitary gland. Often, they are incidentally
found, involving the sellar or suprasellar areas. They often do not contrast enhance but the rim
may show faint enhancement. On T1 and T2 weighted imaging, they are equivocally intense to
hyperintense. On pathology, they are marked by single-layer cuboidal epithelium.
Craniopharyngiomas often avidly enhance and involve the sellar and suprasellar areas. They may
have calcifications in the pediatric adamantinomatous forms or without any calcification in the
adult-predominant papillary form. They also tend to be T2 hyperintense. Pituitary adenomas show
contrast enhancement generally and avidly in the solid portion and are T2 isointense. With
lymphocytic hypohysitis, the pituitary infundibulum is often thickened with areas of parasellar
enhancement. There is also dural based enhancement that can be appreciated; it too is a T2
hypointense phenomenon. Empty sella syndrome will demonstrate a sellar region with a CSF
signal; thus T2 is often isointense to CSF with no enhancement.
References:
Chapman PR, Singhal A, Gaddamanugu S, Prattipati V. Neuroimaging of the Pituitary Gland:
Practical Anatomy and Pathology. Radiol Clin North Am. 2020 Nov;58(6):1115-1133.
Pubmed Web link: https://pubmed.ncbi.nlm.nih.gov/33040852/
Voelker JL, Campbell RL, Muller J. Clinical, radiographic, and pathological features of symptomatic
Rathke’s cleft cysts. J Neurosurg. 1991 Apr;74(4):535-544.

39
Q

An autopsy specimen is shown. Which of the following is the most likely diagnosis?
Answers:
A. Parkinson disease
B. Middle cerebral artery stroke
C. Pick disease
D. Huntington disease
E. Alzheimer disease

A

Alzheimer disease

Discussion:
This gross specimen of the brain shows global degeneration and atrophy of the gyri with
prominence of the sulci. This is most consistent with Alzheimer disease. Selective degeneration of
the caudate nucleus is a hallmark of Huntington’s disease. Pick’s disease, or frontotemporal
dementia, has atrophy and degeneration of the bilateral frontal and temporal lobes. Parkinson’s
disease shows degeneration of the substantia nigra pars compacta. A middle cerebral artery
stroke would not cause atrophy and degeneration, but necrosis and encephalomalacia.
References:
National Institute of Neurological Disorders and Stroke. Alzheimer’s Disease Information Page.
NINDS Web site. Available at: http://www.ninds.nih.gov/disorders/alzheimersdisease
/alzheimersdisease.htm. Updated November 21, 2013.
Arvanitakis Z, Shah RC, Bennett DA. Diagnosis and Management of Dementia: Review. JAMA.
2019 Oct 22;322(16):1589-1599. doi: 10.1001/jama.2019.4782. PMID: 31638686; PMCID:
PMC7462122.
Sengoku R. Aging and Alzheimer’s disease pathology. Neuropathology. 2020 Feb;40(1):22-29. doi:
10.1111/neup.12626. Epub 2019 Dec 21. PMID: 31863504.

40
Q

A 49-year-old man is evaluated because of sudden onset of headache, lethargy, and partial left third nerve palsy. Non-contrast coronal and sagittal MR images are shown. Intravenous administration of which of the following is the most appropriate initial treatment?
Answers:
A. Amicar (Aminocaproic Acid)
B. Nimodipine
C. Gadolinium
D. Aspirin
E. Tissue Plasminogen Activator (TPA)

A

Gadolinium

Discussion:
In pituitary apoplexy, the pituitary shows a heterogeneous appearance with contrast and a
significant proportion will show ring enhancement.
TPA is used in the setting of ischemic strokes and has no role in pituitary apoplexy. Similarly,
aspirin has no role in pituitary apoplexy.
Nimodipine is a calcium channel blocker that can have a role in the management of vasospasm
after subarachnoid hemorrhage, but has no role in pituitary apoplexy.
Aminocaproic Acid is an antifibrinolytic therapy considered for use in patients with aneurysmal
rupture, but this has no role in pituitary apoplexy.
References:
Iqbal F, Adams W, Dimitropoulos I, Muquit S, Flanagan D. Pituitary haemorrhage and infarction:
the spectrum of disease. Endocr Connect. 2021 Feb;10(2):171-179. doi: 10.1530/EC-20-0545.
PMID: 33434143; PMCID: PMC7983520.
Pubmed Web link: https://www-ncbi-nlm-nih-gov.eresources.mssm.edu/pmc/articles
/PMC7983520/
Lee JS, Park YS, Kwon JT, Nam TK, Lee TJ, Kim JK. Radiological apoplexy and its correlation with
acute clinical presentation, angiogenesis and tumor microvascular density in pituitary adenomas. J
Korean Neurosurg Soc. 2011 Oct;50(4):281-7. doi: 10.3340/jkns.2011.50.4.281. Epub 2011 Oct 31.
PMID: 22200007; PMCID: PMC3243828.
Pubmed Web link: https://pubmed-ncbi-nlm-nih-gov.eresources.mssm.edu/22200007/

41
Q

In a patient with atrial fibrillation and congestive heart failure, which of the following risk factors is most predictive of the development of the pathology in the autopsy specimen shown?
Answers:
A. Prior stroke or transient ischemic attack (TIA)
B. Vascular disease history (ie. Myocardial infarction, Peripheral Arterial Disease)
C. Diabetes Mellitus
D. Hypertension
E. Age 65-74

A

Prior stroke or transient ischemic attack (TIA)

Discussion:
Critique:
The autopsy specimen in figure 1 demonstrates encephalomalacia secondary to a prior left Middle
Cerebral Artery (MCA) stroke. Atrial fibrillation is a significant risk factor for the development of
thromboembolic ischemic stroke. The CHA(2)D(2)S-VASc score (see below) is a useful clinical tool
to aid in the decision making for starting anticoagulation therapy for patients with atrial fibrillation.
Individuals with a CHA(2)D(2)S-VASc score ≥2 should be considered for anticoagulation therapy.
Of the seven components of the scoring system, age ≥ 75 and prior history of stroke/TIA are the
strongest risks factor for developing thromboembolic stroke and both are given 2 points.
Age (65-74), hypertension, vascular disease history, and diabetes mellitus are all risk factors for
thromboembolic stroke in atrial fibrillation; however, these factors only contribute 1 point to the
CHA(2)D(2)S-VASc scoring system.
Figure 2: CHA(2)D(2)S-VASc score
- C: CHF (+1)
- A: ≥ 75 (+2)
- D: Diabetes mellitus (+1)
- S: Stroke/TIA history (+2)
- V: Vascular disease (+1)
- A: 65-74 (+1)
- S: Sex, female (+1)
References:
Reference (1)
Gage BF, van Walraven C, Pearce L, Hart RG, Koudstaal PJ, Boode BS, Petersen P. Selecting
patients with atrial fibrillation for anticoagulation: stroke risk stratification in patients taking aspirin.
Circulation. 2004 Oct 19;110(16):2287-92. doi: 10.1161/01.CIR.0000145172.55640.93. Epub 2004
Oct 11. PMID: 15477396.
Pubmed Web link
https://pubmed.ncbi.nlm.nih.gov/15477396/
Reference (2)
Lip GY, Nieuwlaat R, Pisters R, Lane DA, Crijns HJ. Refining clinical risk stratification for predicting
stroke and thromboembolism in atrial fibrillation using a novel risk factor-based approach: the euro
heart survey on atrial fibrillation. Chest. 2010 Feb;137(2):263-72. doi: 10.1378/chest.09-1584.
Epub 2009 Sep 17. PMID: 19762550.
Pubmed Web link
https://pubmed.ncbi.nlm.nih.gov/19762550/

42
Q

The tumor in the contrast-enhanced T1-weighted MR image shown contains which of the following histologic features?
Answers:
A. Flexner-Winterstein Rosettes
B. Adamantinomatous types with distinctive epithelium that forms stellate reticulum, wet keratin, and basal palisades or a Papillary type with fibrovascular cores linked by nonkeratizing squamous epithelium
C. Homer Wright Rosettes
D. Thin-walled capillary vessels, densely packed together with scarce parenchyma
E. Perivascular pseudorosettes

A

Adamantinomatous types with distinctive epithelium that forms stellate reticulum, wet keratin, and basal palisades or a Papillary type with fibrovascular cores linked by non-keratizing squamous epithelium

Discussion:
The tumor in the contrast-enhanced T1-weighted MR image shown is classic for
craniopharyngiomas due to its cystic and solid components as well as its location in the sella and
suprasellar region. Craniopharyngiomas will frequently grow to involve the third ventricle,
hypothalamus, optic chiasm and pituitary gland.
There are two main categories of craniopharyngiomas.
Adamantinomatous craniopharyngioma has a bimodal peak for age at presentation (5-15 and
45-60). On MRI they display cauliflower shape, frequently displaying calcifications, enhancement,
and/or cysts. On pathology, there is a distinctive epithelium that forms stellate reticulum, wet
keratin, and basal palisades.
The papillary craniopharyngioma often manifests in patients 40-55 years old. On MRI, they are
mostly solid and rarely cystic. On histology, there are fibrovascular cores lined by non-keratizing
squamous epithelium.
Thin-walled capillary vessels, densely packed together with scarce parenchyma is seen in
hemangioblastoma.
Perivascular pseudorosettes are often seen in ependymoma and astroblastoma, but can be seen
in many neuroendocrine and neurological tumors.
Homer wright rosettes can be seen in medulloblastomas or PNETs.
Flexner-Winterstein Rosettes are particularly characteristic of retinoblastomas, but can also be
seen in pineoblastomas and medulloepitheliomas.
References:
Müller HL, Merchant TE, Warmuth-Metz M, Martinez-Barbera JP, Puget S. Craniopharyngioma.
Nat Rev Dis Primers. 2019 Nov 7;5(1):75. doi: 10.1038/s41572-019-0125-9. PMID: 31699993.
Pubmed Web link: https://pubmed-ncbi-nlm-nih-gov.eresources.mssm.edu/31699993/
Müller HL. Craniopharyngioma. Handb Clin Neurol. 2014;124:235-53. doi:
10.1016/B978-0-444-59602-4.00016-2. PMID: 25248591.
Pubmed Web link: https://pubmed-ncbi-nlm-nih-gov.eresources.mssm.edu/25248591/

43
Q

The neuronal inclusion body evident in the photomicrograph of the histologic section shown is most likely a
Answers:
A. Lewy body
B. Tachyzoite
C. Neurofibrillary tangle
D. Amyloid beta plaque
E. Cowdry A Body

A

Lewy body

Discussion:
The images shown demonstrate Lewy bodies, which are inclusion bodies – abnormal aggregations
of protein – that develop inside nerve cells in specific diseases. These pathological hallmarks are
found in Lewy Body dementia and Parkinson’s disease dementia and can help distinguish this
disease from other types of dementia, such as Alzheimer’s disease.
Neurofibrillary tangles are cytoplasmic paired helical filaments associated with Alzheimer disease.
Amyloid beta plaques are extracellular and associated with Alzheimer disease.
Cowdry A bodies are intranuclear eosinophilic masses associated with herpes virus infection (HSV,
CMV).
Tachyzoites are extracellular infective form of toxoplasma gondii.
References:
Wakabayashi K, Tanji K, Odagiri S, Miki Y, Mori F, Takahashi H. The Lewy body in Parkinson’s
disease and related neurodegenerative disorders. Mol Neurobiol. Apr 2013;47(2):495-508.
Pubmed Web link: doi:10.1007/s12035-012-8280-y
Hansen D, Ling H, Lashley T, Holton JL, Warner TT. Review: Clinical, neuropathological and
genetic features of Lewy body dementias. Neuropathol Appl Neurobiol. Dec 2019;45(7):635-654.
Pubmed Web link: doi:10.1111/nan.12554

44
Q

CD68 staining of the biopsy specimen from the corpus callosum of a 30-year-old woman is shown. Which of the following therapies is indicated?
Answers:
A. Idarucizumab
B. Riluzole
C. Temozolomide
D. Bevacizumab
E. Ocrelizumab

A

Ocrelizumab

Discussion:
CD68 is a marker of macrophages, which can be found as shown as part of demyelinating disease
in multiple sclerosis (MS). MS lesions evolve differently during early versus chronic disease
phases, and within each phase, different plaque types and plaques in different stages of
demyelinating activity are evident. Histologically, several basic processes drive the formation of
plaques: inflammation, myelin breakdown, astrogliosis, oligodendrocyte injury, neurodegeneration,
axonal loss, and remyelination.
A combination of histologic and/or immunohistochemical stains can be used to visualize these
processes and to neuropathologically diagnose inflammatory demyelinating lesions as consistent
with MS: hematoxylin and eosin stain (demonstrates tissue and cell morphology), myelin stains
(Luxol fast blue/periodic acid-Schiff, Luxol fast blue/hematoxylin/eosin, or immunohistochemistry
for myelin proteins), macrophage-specific markers (immunohistochemistry for KiM1P or CD68),
stains for axons (Bielschowsky silver impregnation or immunohistochemistry for neurofilament
protein), stains for astrocytes (hematoxylin and eosin or immunohistochemistry for glial fibrillary
acidic protein), and stains for the different lymphocyte subtypes (immunohistochemistry for CD3,
CD4, CD8, CD20, and/or CD138).
Once you recognize that the question stem is asking to identify MS as the disease diagnosed, the
selection of Ocrelizumab is then made, as this is the monoclonal antibody that targets CD20 on B
cells in the treatment of MS.
Riluzole blocks glutamatergic neurotransmission in the treatment of ALS.
Bevacizumab is an anti-VEGF molecule that reduces vascularization in the treatment of GBM.
Idarucizumab is a monoclonal antibody fragment that binds dabigatran to reverse the effects of
anticoagulation.
Temozolomide is an alkylating agent utilized in the treatment of GBM.
References:
Hauser SL, Cree BAC. Treatment of Multiple Sclerosis: A Review. Am J Med. Dec
2020;133(12):1380-1390 e2. doi:10.1016/j.amjmed.2020.05.049
Yamout B, Sahraian M, Bohlega S, et al. Consensus recommendations for the diagnosis and
treatment of multiple sclerosis: 2019 revisions to the MENACTRIMS guidelines. Mult Scler Relat
Disord. Jan 2020;37:101459. doi:10.1016/j.msard.2019.101459
Kuhlmann T, Lassmann H, Bruck W. “Diagnosis of inflammatory demyelination in biopsy
specimens: a practical approach” Acta Neuropathol. 2008 Mar; 115(3):275-87.

45
Q

A 41-year-old Marine presents 18 months after having suffered a concussion from an explosion
outside his armored personnel carrier. He complains of delayed onset of progressive upper back
and right chest wall pain. He has a low thoracic sensory level and profound weakness in his hip
flexors with a spastic gait and sustained clonus. MR images without contrast are shown. This
lesion is most consistent with which of the following diagnoses?
Answers:
A. Ependymoma
B. Hemangioblastoma
C. Traumatic Arachnoid Scar
D. Transverse Myelitis
E. Spinal Cord Hemisection

A

Traumatic Arachnoid Scar

Discussion:
The lesion is most consistent with post-traumatic arachnoiditis causing spinal cord tethering. The
history lends itself to a post-traumatic etiology for this lesion as opposed to a neoplastic or
demyelinating process. Moreover, the delayed and progressive nature of the lesion decreases the
likelihood of an acute demyelinating process. The CSF isointense signal of the lesion reinforces
the scarring encountered with adhesive arachnoiditis. Neoplastic lesions are often of differing T2
signal intensities based on the composition of the lesion. Although post-traumatic, there is no
radiographic evidence of a sectioned spinal cord.
References:
Jurga S, Szymańska-Adamcewicz O, Wierzchołowski W, Pilchowska-Ujma E, Urbaniak Ł. Spinal
adhesive arachnoiditis: three case reports and review of literature. Acta Neurol Belg. 2021
Feb;121(1):47-53.
Pubmed Web link: https://pubmed.ncbi.nlm.nih.gov/32833147/
Wright MH, Denney LC. A comprehensive review of spinal arachnoiditis. Orthop Nurs. 2003 MayJun;22(3):215-9; quiz 220-1.
Pubmed Web link: https://pubmed.ncbi.nlm.nih.gov/12803151/

46
Q

Histologic examination of the autopsy specimen shown is most likely to demonstrate which of the following?
Answers:
A. Negri body
B. Lewy body
C. Amyloid plaque
D. Pick body
E. Bunina body

A

Pick body

Discussion:
This gross specimen of the brain shows atrophy and degeneration of the frontal and temporal
lobes, most consistent with Pick disease (frontotemporal dementia). Histologically, Pick disease is
defined by the presence of the Pick body, which are spherical cytoplasmic inclusions that have tau
protein and ubiquitin. Lewy body dementia and Parkinson’s disease both have the presence of
intracellular Lewy bodies, which are spherical and associated with a halo and consist of alphasynuclein; however, these two pathologies are not associated with frontotemporal atrophy.
Alzheimer disease has extracellular amyloid plaques containing amyloid beta protein. However,
Alzheimer disease would show global degeneration and atrophy of the gyri with prominence of the
sulci. Rabies produces intracellular Negri bodies within the hippocampus and cerebellum
consisting of viral proteins, and Amyotrophic lateral sclerosis produces spherical Bunina bodies,
consisting of ubiquitin. However, neither of these disease processes are associated with
frontotemporal atrophy.
References:
Kertesz A. Progress in clinical neurosciences: Frontotemporal dementia-pick’s disease. Can J
Neurological Sci. 2006 May 33(2):141-8.
Kertesz A, Munoz D. Relationship between frontotemporal dementia and corticobasal
degeneration/progressive supranuclear palsy. Dement Geriatr Cogn Disord. 2004;17(4):282-6. doi:
10.1159/000077155. PMID: 15178937.

47
Q

The most likely diagnosis for the lesion from which the microscopic specimen shown was obtained is
Answers:
A. Subacute combined degeneration
B. Amyotrophic lateral sclerosis
C. Brown-Sequard Syndrome
D. Tabes Dorsalis
E. Ependymoma

A

Amyotrophic lateral sclerosis

Discussion:
This microscopic specimen shows deterioration of the anterior horn cells as well as the lateral
corticospinal tracts and anterior corticospinal tracts, which are contrasted nicely to the preservation
of the dorsal columns. Amyotrophic lateral sclerosis affects both upper and lower motor neurons,
causing degradation of the corticospinal tracts as well as anterior horn cells. Subacute combined
degeneration, caused vitamin B12 deficiency, and Tabes Dorsalis, caused by neurosyphilis, are
defined by degradation of the dorsal columns. An ependymoma is a type of tumor, which would
present as a mass lesion. Brown-Sequard Syndrome is due to hemisection of the spinal cord,
which would produce a lesion on either the left or right side of the microscopic section.
References:
Saberi S, Stauffer JE, Schulte DJ, Ravits J. Neuropathology of Amyotrophic Lateral Sclerosis and
Its Variants. Neurol Clin. 2015;33(4):855-876. doi:10.1016/j.ncl.2015.07.012
Piao YS, Wakabayashi K, Kakita A, Yamada M, Hayashi S, Morita T, Ikuta F, Oyanagi K, Takahashi
H. Neuropathology with clinical correlations of sporadic amyotrophic lateral sclerosis: 102 autopsy
cases examined between 1962 and 2000. Brain Pathol. 2003 Jan;13(1):10-22. doi:
10.1111/j.1750-3639.2003.tb00002.x. PMID: 12580541; PMCID: PMC8095891.

48
Q

Which of the following is most likely to be found in the brain of a patient who died of long-standing hepatic insufficiency with jaundice?
Answers:
A. Neuritic Plaque
B. Alzheimer Type II Astrocyte
C. Neurofibrillary Tangle
D. Hirano Body
E. Bunina Body

A

Alzheimer Type II Astrocyte

Discussion:
Alzheimer type II astrocytes are non-neuronal glial cells associated with hepatic encephalopathy.
The exact pathology remains unknown but is commonly identified in diseases that involve the
accumulation of ammonia, such as liver disease and Wilson’s disease. Alzheimer’s type II
astrocytes are not found in Alzheimer’s disease, but are named as such because they were
identified by the same neuroscientist who also described Alzheimer’s disease (AD), Alois
Alzheimer.
Neurofibrillary tangles are associated with tau which are found in AD and sometimes Parkinson’s
Disease.
Neuritic plaques are associated with amyloid beta in AD.
Hirano (Bunina) bodies stain positive for actin and can be found in nerves in neurodegenerative
disease, such as amyotrophic lateral sclerosis, Alzheimer’s disease, and Creutzfelt-Jacob disease.
References:
Norenberg MD. The role of astrocytes in hepatic encephalopathy. Neurochem Pathol. 1987 FebApr;6(1-2):13-33. doi: 10.1007/BF02833599. PMID: 3306480.
Agarwal AN, Mais DD. Sensitivity and Specificity of Alzheimer Type II Astrocytes in Hepatic
Encephalopathy. Arch Pathol Lab Med. Oct 2019;143(10):1256-1258.
doi:10.5858/arpa.2018-0455-OA

49
Q

The lesion evident in the photograph shown is most likely the result of which of the following?
Answers:
A. Amyloid angiopathy
B. Neoplasm
C. Duret hemorrhage
D. Vasculitis
E. Hemorrhagic conversion of ischemic stroke

A

Duret hemorrhage

Discussion:
Brainstem hemorrhages can be classified as primary or secondary. Causes of primary brainstem
hemorrhage includes direct laceration or contusion, penetrating injury, shearing injuries, and
petechial hemorrhages. Secondary brainstem hemorrhages in cranial trauma victims occur at a
later stage from descending transtentorial herniation, known as Duret hemorrhages. They are
typically found in the lower mesencephalon and ventral pons. The pathophysiology remains
controversial. Although most authors favor an arterial origin (stretching and laceration of
perforating branches of the basilar artery), there are equally strong arguments for a venous origin
(thrombosis and venous infarction). Historically, Duret hemorrhages have conferred a devastating
prognosis. However, recent case reports suggest that cognitive and functional recovery are
possible after Duret hemorrhages.
Primary CNS vasculitis demonstrates transmural inflammation with vessel wall injury on pathologic
examination. Brainstem hemorrhages are not common.
Cerebral amyloid angiopathy (CAA) is characterized by the accumulation of amyloid fibrils in the
walls of small- to medium-sized arterial blood vessels, and in capillaries of the CNS parenchyma
and leptomeninges, and is a major cause of spontaneous intracerebral hemorrhage in the elderly.
The most common clinical presentations of CAA are ICHs, which occur predominantly in lobar
sites, not the brainstem.
Hemorrhagic transformation (HT) is a common complication in patients with acute ischemic stroke.
It occurs when peripheral blood extravasates across a disrupted blood brain barrier into the brain
following ischemic stroke. It most often results in petechial hemorrhages in strokes involving the
cortex.
Given the appearance and location of this bleed, this is unlikely to be due to CNS vasculitis, CAA,
hemorrhagic conversion, or an underlying neoplasm.
References:
Parizel PM, Makkat S, Jorens PG, et al. Brainstem hemorrhage in descending transtentorial
herniation (Duret hemorrhage). Intensive Care Med. 2002;28(1):85-88.
Pubmed Web link: https://pubmed.ncbi.nlm.nih.gov/11819006/
Edlow BL, Threlkeld ZD, Fehnel KP, Bodien YG. Recovery of Functional Independence After
Traumatic Transtentorial Herniation With Duret Hemorrhages. Front Neurol. 2019;10:1077.
Published 2019 Oct 9.
Pubmed Web link: https://www.ncbi.nlm.nih.gov/pmc/articles/PMC6794605/

50
Q

A 46-year-old man is brought to the emergency department because of a three-day history of headaches, lethargy, and worsening confusion. Analysis of cerebrospinal fluid shows 1500/mm3 mononuclear white blood cells; glucose level is within the reference range. Photomicrographs of a
brain biopsy specimen are shown. Which of the following is the most likely diagnosis?
Answers:
A. Creutzfeldt-Jakob disease
B. Subacute sclerosing panencephalitis
C. Herpes Simplex encephalitis
D. Japanese encephalitis virus
E. HIV subacute encephalitis

A

Herpes Simplex encephalitis

Discussion:
Herpes simplex virus (HSV) type 1 and 2 are DNA viruses, belonging to the herpes virus family.
These groups of viruses, in addition to primary infection, are known to establish latency in sensory
ganglia and cause recurrent disease manifesting as cutaneous eruptions by reactivation in an
immunocompromised host. These viruses may manifest as commonly recognized cold sores, or
less frequently as herpetic keratitis, presumably related to the latency in trigeminal ganglia. The
pathogenetic association between latent HSV infections and HSV encephalitis is not well
understood. The encephalitis caused by HSV-1 in children and adults is much more circumscribed
and is limited to inferior frontal, medial temporal and adjacent insular cortex, and cingulate gyrus,
while sparing the parieto-occipital lobes and cerebellum. HSV encephalitis in children and adults is
the most frequent nonendemic cause of acute necrotizing encephalitis. CSF PCR for HSV-1 and
HSV-2 is the diagnostic test of choice. Immediate administration of intravenous acyclovir is
recommended if diagnosis is suspected. Clinical presentation is typically as described in this
patient, with altered mental status (typically for ≥ 24 h). This is accompanied by evidence of brain
parenchymal inflammation. Findings supportive of brain inflammation may include fever, new
seizures, focal neurologic signs, cerebrospinal fluid (CSF) pleocytosis, and radiological and/or
EEG abnormalities.
CSF analysis typically demonstrates lymphocytosis, with or without red blood cells or
xanthochromia, mildly elevated protein and normal or mildly decreased glucose. Brain biopsies
may be performed when diagnosis remains uncertain. Pathology demonstrates polymorphs and
lymphocytes accumulated around the blood vessels. The neurons, glia, and vascular endothelial
cells look hypereosinophilic, and intranuclear eosinophilic inclusions in cells are visible at the edge
of lesions. Occasional areas of active microglial nodules with neuronophagia may be seen as a
feature of viral invasion. In hemorrhagic lesions, affected regions contain sheets of necrotic cells
admixed with gitter cells (seen on the right-hand image) and intense perivascular and interstitial
lymphocytic infiltrates and apoptotic nuclei.
Subacute encephalitis is characterized by demyelination, gliosis of the gray and white matter, focal
necrosis, microglial nodules, atypical oligodendrocyte nuclei, and multinucleated cells. Subacute
sclerosing panencephalitis is caused by the measles virus and demonstrates neuronal loss,
reactive astrocytosis, large areas of white matter demyelination, and focal microglial responses on
pathology. Japanese encephalitis virus (JEV) typically has a triad of perivascular lymphocytic
cuffing, microglial nodules, and pale spongy lesions with histiocytic reactions. Creutzfeldt-Jakob
disease has neuronal loss, spongiform degeneration, reactive astrogliosis, and deposition of
misfolded PrP species on pathology.
References:
Bradshaw MJ, Venkatesan A. Herpes Simplex Virus-1 Encephalitis in Adults: Pathophysiology,
Diagnosis, and Management. Neurotherapeutics. 2016;13(3):493-508.
Pubmed Web link: https://pubmed.ncbi.nlm.nih.gov/27106239/
Shankar SK, Mahadevan A, Kovoor JM. Neuropathology of viral infections of the central nervous
system. Neuroimaging Clin N Am. 2008;18(1):19-vii.
Pubmed Web link: https://pubmed.ncbi.nlm.nih.gov/18319153/

51
Q

The histologic slide from the hippocampus shown was stained with modified Bielschowsky silver stain. Which of the following is the most likely diagnosis?
Answers:
A. Amyotrophic lateral sclerosis
B. Pick disease
C. Alzheimer disease
D. Parkinson disease
E. Rabies

A

Pick disease

Discussion:
Pick bodies are found in Pick disease, or frontotemporal dementia. They are spherical cytoplasmic
inclusions that have tau protein and ubiquitin. The Bielschowsky silver stain is used to detect
nerves and nerve fibers, especially in the workup of neurodegenerative disorders. Pick disease is
known to affect the hippocampus, consistent with the location of this lesion. Alzheimer disease
also affects the hippocampus and has intracellular inclusions, neurofibrillary tangles and Hirano
bodies, but they are elongated and not spherical. Parkinson’s disease has Lewy bodies, which are
also spherical, but are associated with a halo and are not normally found in the hippocampus.
Rabies produces intracellular Negri bodies within the hippocampus and cerebellum, consisting of
viral proteins, but they are much smaller than Pick bodies. Amyotrophic lateral sclerosis produces
spherical Bunina bodies, consisting of ubiquitin, but they are found in ventral horn cells.
References:
Munoz DG, Dickson DW, Bergeron C, Mackenzie IR, Delacourte A, Zhukareva V. The
neuropathology and biochemistry of frontotemporal dementia. Ann Neurol. 2003;54 Suppl
5:S24-248.
Josephs KA, Holton JL, Rossor MN, Godbolt AK, Ozawa T, Strand K, Khan N, Al-Sarraj S, Revesz
T. Frontotemporal lobar degeneration and ubiquitin immunohistochemistry. Neuropathol Appl
Neurobiol. 2004 Aug;30(4):369-73. doi: 10.1111/j.1365-2990.2003.00545.x. PMID: 15305982

52
Q

Lewy bodies and Lewy dendrites in the brain tissue of patients with Parkinson disease contain aggregates of which of the following proteins?
Answers:
A. Amyloid beta
B. Superoxide dismutase
C. Alpha synuclein
D. Tau protein
E. Huntingtin Protein

A

Alpha synuclein

Discussion:
One of the diagnostic hallmarks of Parkinson’s Disease is the accumulation of the protein alphasynuclein in the brain, which are the main components of Lewy Bodies. This differentiates it from
Alzheimer disease, where there is accumulation of intracellular tau protein and extracellular
amyloid beta. In familial Amyotrophic Lateral Sclerosis, the mutated superoxide dismutase (SOD1)
protein accumulates, creating Bunina Bodies. Huntington’s Disease has accumulation of Huntingtin
protein.
References:
Brundin P. Prion-like transmission of protein aggregates in neurodegenerative diseases. Nature
Reviews: Molecular Cell Biology. 2010; 11: 301-307.
Ross CA, Poirier MA. Protein aggregation and neurodegenerative disease. Nat Med. 2004 Jul;10
Suppl:S10-7. doi: 10.1038/nm1066. PMID: 15272267.

53
Q

The patient whose MR image with gadolinium is shown most likely has which of the following conditions?
Answers:
A. Neurofibromatosis Type 1
B. Hereditary Hemorrhagic Telangiectasias
C. Familial Cerebral Cavernous Malformation Type 1
D. von Hippel Lindau
E. Familial Cerebral Cavernous Malformation Type 2

A

von Hippel Lindau

Discussion:
The patient whose MR image with gadolinium is shown most likely has von Hippel Lindau
syndrome. This is an autosomal dominant disorder of the vHL gene located on chromosome 3.
The condition is associated with hemangioblastomas (shown in the MRI) as well as systemic cystic
lesions such as pheochromocytoma, renal cell carcinoma and pancreatic cysts, among others.
Hemangioblastomas often occur in the posterior fossa and/or the spinal cord and have a nonenhancing cystic lesion with an avidly enhancing mural nodule.
Familial Cerebral Cavernous Malformations Type 1 and 2 lead to the development of cavernomas,
while Neurofibromatosis Type 1 can lead to gliomas, neither of which has this pattern of MRI
finding. Hemorrhagic hereditary telangiectasis, also known as Osler-Weber-Rendu syndrome,
causes formations of arteriovenous malformations, which are also characteristically different from
this cystic lesion with an enhancing nodule.
References:
Frantzen C, Klasson TD, Links TP, and Giles RH. Von Hippel-Lindau Syndrome. GeneReviews. August 6, 2015
Pubmed Web link: http://www.ncbi.nlm.nih.gov/books/NBK1463/
Awad IA, Polster SP. Cavernous Angiomas: Deconstructing a Neurosurgical Disease. J Neurosurg.
2019 Jul 1;131(1):1-13. Pubmed Web link: https://pubmed.ncbi.nlm.nih.gov/31261134/

54
Q

A 66-year-old man dies from an intracerebral hemorrhage. He has a history of ischemic events
since age 20 years in a staccato-type pattern of transient ischemic attacks, strokes, and then
stabilization. Histologic examination of his middle cerebral artery is shown. The intima is indicated
by the arrow. The H & E-stained histopathology figure is most consistent with which of the
following diagnoses?
Answers:
A. Moyamoya
B. Intracranial atherosclerosis
C. Intracranial saccular aneurysm
D. Arteriovenous malformation
E. Dural arteriovenous fistula

A

Moyamoya

Discussion:
Critique:
The histopathology slide demonstrates hyperproliferation of an arterial vessel wall with occlusion of
the lumen without arteriosclerotic changes, consistent with moyamoya disease. Moyamoya is
characterized by progressive steno-occlusion of the bilateral distal internal carotid arteries (ICA)
and proximal middle cerebral arteries (MCA). There is a compensatory dilation of small collateral
vessels in the region that has been described as appearing like “moyamoya,” which is Japanese
for “puff of smoke,” on catheter angiography. Moyamoya has a bimodal age distribution for
presentation in children and adults in their 40’s. The most common presentation is recurrent
ischemia, which is often described as a staccato-type pattern of transient ischemic attacks,
strokes, and then stabilization. However, intracerebral hemorrhage is also common in these
patients.
Intracranial atherosclerosis may have a similar presentation as moyamoya with recurrent ischemic
events. However, the age of onset is typically in the elderly unlike this patient who initially
presented in his 20s. Additionally, histopathology will demonstrate atheromatous plaques.
Intracranial saccular aneurysms, arteriovenous malformations, and dural arteriovenous fistulas can
all result in intracerebral hemorrhage. However, these conditions are much less likely to be
associated with a staccato-type pattern of transient ischemic attacks, strokes, and then
stabilization.
References:
Reference (1)
Scott RM, Smith ER. Moyamoya disease and moyamoya syndrome. N Engl J Med. 2009 Mar
19;360(12):1226-37. doi: 10.1056/NEJMra0804622. PMID: 19297575.
Pubmed Web link
https://pubmed.ncbi.nlm.nih.gov/19297575/
Reference (2)
Smith ER, Scott RM. Moyamoya: epidemiology, presentation, and diagnosis. Neurosurg Clin N
Am. 2010 Jul;21(3):543-51. doi: 10.1016/j.nec.2010.03.007. PMID: 20561502.
Pubmed Web link
https://pubmed.ncbi.nlm.nih.gov/20561502/

55
Q

A 32-year-old woman is evaluated for new-onset paresthesias in the left hand and arm. MR imaging shows a 2 × 2-cm ring-enhancing lesion in the right parietal lobe. A stereotactic biopsy is performed. Which of the following pathologic findings is most likely to support a diagnosis of tumefactive demyelination?
Answers:
A. CD-68 positive macrophages
B. Positive for EGFR mutation
C. Nuclear atypia
D. Microvascular proliferation
E. Pseudopalisading necrosis

A

CD-68 positive macrophages

Discussion:
CD68+ macrophages are consistent with tumefactive demyelination and the disease process
found pathologically in MS. CD68 is a marker of macrophages, which can be found as shown as
part of demyelinating disease in multiple sclerosis (MS). MS lesions evolve differently during early
versus chronic disease phases, and within each phase, different plaque types and plaques in
different stages of demyelinating activity are evident. Histologically, several basic processes drive
the formation of plaques: inflammation, myelin breakdown, astrogliosis, oligodendrocyte injury,
neurodegeneration, axonal loss, and remyelination.
Distinguishing the diagnosis of tumefactive MS from other ring enhancing lesions can be difficult,
and may require pathology confirmation from a biopsy unless other signs of demyelinating disease
are present. The differential diagnosis is broad, but can include neoplastic disease, cerebral
abscesses and other infectious etiologies.
Microvascular proliferation, pseudopalisading necrosis, nuclear atypia, and EFGR mutations are
pathological findings identified with GBM.
References:
Fallah A, Banglawala S, Ebrahim S, Paulseth JE, Jha NK. Case Series: tumefactive demyelinating
lesions: a diagnostic challenge. Can J Surg. Feb 2010;53(1):69-70.
Aldape K, Zadeh G, Mansouri S, Reifenberger G, von Deimling A. Glioblastoma: pathology,
molecular mechanisms and markers. Acta Neuropathol. Jun 2015;129(6):829-48.
Pubmed Web link: doi:10.1007/s00401-015-1432-1

56
Q

Progressive multifocal leukoencephalopathy is believed to be the result of which of the following?
Answers:
A. Infectious mononucleosis
B. Reactivation of latent virus from sensory ganglia
C. Viral transmission through mosquitoes, birds, and ticks
D. Transplacental viral spread
E. Opportunistic viral infection

A

Opportunistic viral infection

Discussion:
Progressive multifocal leukoencephalopathy (PML) is the result of an opportunistic viral infection
caused by the polyoma virus group, JC virus, which specifically involves the white matter of the
brain. The incidence of PML increased dramatically with the advent of AIDS, reflecting its
opportunistic nature, but has also declined with the initiation of highly active antiretroviral therapy
(HAART) for AIDS.
JC virus is a double stranded DNA virus. Oligodendrocytes in the white matter support productive
JC virus infection, which leads to cell lysis and demyelination. PML essentially is restricted to
immunocompromised cases, but it rarely can be found in patients who do not have an identifiable
underlying disease. It manifests with diverse neurologic disorders depending on the anatomic sites
of the nervous system involved. Routine CSF analysis is normal. PCR analysis for JC viral nucleic
acid is a sensitive and specific means of diagnosis.
The brain on gross examination is normal. But on slicing, multiple confluent foci of grayish
discoloration can be found in the white matter or gray-white junction. With progression, the
involved white fiber tracts look granular or pitted and are very discernable on MRI.
Reactivation of latent virus from sensory ganglia is seen in HSV type 1 and 2 infections. Infectious
mononucleosis is associated with Epstein-Barr virus. Rubella and CMV are spread
transplacentally. PML is not associated with viral transmission through birds, mosquitoes, or ticks.
Flaviviruses such as Japanese encephalitis virus, West Nile virus, and St. Louis encephalitis virus
can be transmitted through birds, mosquitoes, and ticks.
References:
Shankar SK, Mahadevan A, Kovoor JM. Neuropathology of viral infections of the central nervous
system. Neuroimaging Clin N Am. 2008;18(1):19-vii.
Pubmed Web link: https://pubmed.ncbi.nlm.nih.gov/18319153/
Cortese I, Reich DS, Nath A. Progressive multifocal leukoencephalopathy and the spectrum of JC
virus-related disease. Nat Rev Neurol. 2021;17(1):37-51.
Pubmed Web link: https://pubmed.ncbi.nlm.nih.gov/33219338/

57
Q

Detection of Lewy bodies and Lewy neurites is best achieved by staining for which of the following proteins?
Answers:
A. Ubiquitin
B. Alpha Synuclein
C. Amyloid Beta
D. Actin
E. Tau

A

Alpha Synuclein

Discussion:
Alpha synuclein staining is most specific to Lewy bodies and neurites. Lewy bodies are the
inclusion bodies – abnormal aggregations of protein – that develop inside nerve cells in specific
diseases. These pathological hallmarks are found in Lewy Body dementia and Parkinson’s disease
dementia and can help distinguish this disease from other types of dementia, such as Alzheimer’s
disease.
Neurofibrillary tangles stain positive for Tau. Neuritic plaques stain positive for amyloid beta. Both
are common in Alzheimer’s disease.
Lewy bodies and neurites can also stain positive for ubiquitin, but this is nonspecific and may be
seen in other pathologies such as tangles, and plaques.
Hirano (Bunina) bodies stain positive for actin and can be found in nerves in neurodegenerative
disease, such as ALS, Alzheimer’s disease, and Creutzfelt-Jacob disease.
References:
Ross CA, Poirier MA. Protein aggregation and neurodegenerative disease. Nat Med. 2004 Jul;10
Suppl:S10-7. doi: 10.1038/nm1066. PMID: 15272267.
Gomperts SN. Lewy Body Dementias: Dementia With Lewy Bodies and Parkinson Disease
Dementia. Continuum (Minneap Minn). Apr 2016;22(2 Dementia):435-63.
doi:10.1212/CON.0000000000000309

58
Q

A 70-year-old man comes to the emergency department with complaints of sudden onset of visual
difficulties. His examination is significant for a partial right homonymous hemianopia. An emergent
CT brain scan was obtained (Figure 1). MR imaging of the brain was subsequently obtained, and a
representative axial diffusion weighted sequence image is shown (Figure 2). Which of the following
is the most likely diagnosis?
Answers:
A. Multiple Sclerosis
B. Cerebral Abscess
C. Epidermoid Tumor
D. Herpes Encephalitis
E. Acute Cerebral Ischemia

A

Acute Cerebral Ischemia

Discussion:
The most likely diagnosis is acute cerebral ischemia. Acute infarcts demonstrate areas of
hypodensity of the affected brain tissue on computed tomography and are diffusion restricting on
diffusion weighted imaging. The patient’s age favors this diagnosis as well.
Multiple sclerosis does not have typical finds on CT scan and on MRI often shows multiple areas
of white matter T2 hyperintensity. Although cerebral abscesses and epidermoid tumors both
diffusion-restrict, the available imaging do not demonstrate delineated masses as would be
expected with those pathologies. Herpes encephalitis often involves the mesial temporal lobe
without any acute findings on a CT scan.
References:
El-Koussy M, Schroth G, Brekenfeld C, Arnold M. Imaging of acute ischemic stroke. Eur Neurol.
2014;72(5-6):309-16.
Pubmed Web link: https://pubmed.ncbi.nlm.nih.gov/25323674/
Muir KW, Buchan A, von Kummer R, Rother J, Baron JC. Imaging of acute stroke. Lancet Neurol.
2006 Sep;5(9):755-68. Pubmed Web link: https://pubmed.ncbi.nlm.nih.gov/16914404/

59
Q

Which of the following is the most likely pathological finding in an autopsy specimen from the brain of a patient with Parkinson disease?
Answers:
A. Pick bodies
B. Neurofibrillary tangles
C. Lewy bodies
D. Bunina body
E. Negri body

A

Lewy bodies

Discussion:
Lewy body dementia and Parkinson’s disease both have the presence of intracellular Lewy bodies,
which are spherical and associated with a halo and consist of alpha-synuclein. Pick disease
(frontotemporal dementia) is defined by the presence of the Pick body, which are spherical
cytoplasmic inclusions that have tau protein and ubiquitin. Alzheimer disease is noted
pathologically by intracellular neurofibrillary tangles, which are elongated inclusions containing tau
protein and ubiquitin. Rabies produces intracellular Negri bodies within the hippocampus and
cerebellum consisting of viral proteins, and Amyotrophic lateral sclerosis produces spherical
Bunina bodies, consisting of ubiquitin.
References:
Kalia LV, Lang AE. Parkinson’s disease. Lancet. 2015;386(9996):896-912.2.
Gomez-Nicola D, Perry VH. Microglial dynamics and role in the healthy and diseased brain: a
paradigm of functional plasticity. Neuroscientist. 2015 Apr;21(2):169-84.3
Ellison D, Love S, Chimelli LMC, et al, eds. Neuropathology: a reference text of CNS pathology.
3rd ed. Elsevier Health Science, 2012.

60
Q

A 48-year-old man is evaluated for progressive low back, flank, and proximal right leg dysesthesias. He is neurologically intact, and his urodynamic studies are normal. MR images with and without contrast are shown. The lesion has increased in size by 25% compared with imaging studies performed three years ago. It is most consistent with which of the following diagnoses?
Answers:
A. Meningioma
B. Ganglioglioma
C. Schwannoma
D. Dermoid Cyst
E. Myxopapillary Ependymoma

A

Schwannoma

Discussion:
The most likely diagnosis is a schwannoma of the cauda equina. These lesions arise off the nerve
sheath and are avidly enhancing lesions, often demonstrating thrombosis, fatty degeneration and
cystic changes which distinguish them from neurofibromas. T2 mixed hyperintensity is common.
Myxopapillary ependymomas are often seen at the conus with a typical “sausage-like”
appearance. They are prone to hemorrhage and will often demonstrate a cap-sign on T2 imaging;
they are avidly contrast enhancing. Spinal meningiomas often demonstrate a dural based tail on
contrast-enhanced imaging. Gangliogliomas of the cauda equina are rare tumors that show patchy
enhancement with slow indolent growth. Dermoid cysts in this location are also rare, with a nonenhancing, T2 hyperintense cystic structure at the base of the conus.
References:
Koeller KK, Shih RY. Intradural Extramedullary Spinal Neoplasms: Radiologic-Pathologic
Correlation. Radiographics. 2019 Mar-Apr;39(2):468-490.
Pubmed Web link: https://pubmed.ncbi.nlm.nih.gov/30844353/
Bennett SJ, Katzman GL, Roos RP, Mehta AS, Ali S. Neoplastic cauda equina syndrome: a
neuroimaging-based review. Pract Neurol. 2016 Feb;16(1):35-41.
Pubmed Web link: https://pubmed.ncbi.nlm.nih.gov/26442520